MEDBULLETS VIGNETTES UROLOGY

¡Supera tus tareas y exámenes ahora con Quizwiz!

A 40-year-old male visits a urologist and reports that for the past 2 weeks, his penis has been gradually curving to the right with associated pain during intercourse. He is able to have a normal erection and he does not recollect of any trauma to his penis. Although he is married, he admits to having unprotected sexual relationship with several females in the past year. His vitals are normal and physical examination in unremarkable except for a lesionless curved penis. It is painless to touch. Test results for sexually transmitted disease is pending. Which of the following is the most likely cause? A. Fibrosis of corpus cavernosa B. Congenital hypospadias C. Syphilitic chancre D. Fibrosis of tunica albuginea E. Hypertrophy of corpus cavernosa

suffering from Peyronie disease. Due to fibrosis of tunica albuginea, the patient should be consulted on the use of medical or surgical therapy by a urologist. Peyronie's is an acquired disease stemming from fibrosis of tunica albuginea layer of the penile tissue and is seen in adult males as young as 18 years of age. Although it is thought to be due to trauma to the penis during sexual activity, patients often are not able to recollect any recent trauma to the region of interest. Although the curvature may correct itself spontaneously in some, majority of the patient need medical or surgical intervention. While there are many medical therapies such as collagenase clostridium histolyticum injections and interferon-alpha-2b, they do not reliably treat the fibrosis. Fitkin and Ho report that Peyronie disease most often affects males between the ages of 45 and 60 years old. While many do resolve without treatment, urologists recommend vitamin D, potassium aminobenzoate, and corticosteroid injections as first line medical therapy. While surgery is recommended if conservative methods fail, they do have excellent outcomes. Levy et al. report of a clinical trial using placental matrix-derived mesenchymal stem cells (PM-MSC) for treatment of Peyronie disease. Of 10 plaques injected with PM-MSC, 7 disappeared by third month of trial. In addition, a metric of erectile function, peak systolic velocity, was significantly increased in the cohort. Although a larger sample size is needed, Levy et al. argue that PM-MSC may be beneficial as an effective therapy for Peyronie disease. Incorrect Answers: Answer 1 and 5: Corpus cavernosa is an erectile tissue surrounded by tunica albuginea and is not directly affected. Answer 2: Hypospadias results from fetal tissue malformation of the urethra and is not acquired as an adult. Also, there is no pain associated upon sexual intercourse. Answer 3: Chancre is manifestation of primary syphilis infection and is painless. It has a distinctive lesion on the penis and often presents itself 21 days after exposure. Although our patient is promiscuous, there are no lesions on his penis that would make us think of syphilis.

A 6-month old boy is brought to the pediatrician's office for evaluation of a "lump" in his genital area. His birth history is unremarkable and he has been feeding and eating well. His parents report no other concerns. On physical exam, there is a tense and non-tender mass in the scrotum. The transillumination test is positive. The parents are counseled about his condition and reassured that this will most likely resolve spontaneously.

Hydrocele (snapshot)

A 30-year-old gentleman presents complaining of an abnormal curvature in his penis particularly when he has an erection. He denies symptoms of dyuria and is otherwise in good health. He states that engaging in intercourse has been painful for his wife.

Peyronie Disease(SNAPSHOT)

A 58-year-old man presents to his primary care physician for an annual examination. He does not have any acute complaints and feels otherwise healthy. He has a past medical history of hypertension that is adequately controlled with lisinopril. Family history is remarkable for prostate cancer in his father. Digital rectal examination is notable for an asymmetric, nodular prostate. Prostate-specific antigen level is elevated at 15 ng/mL.

Prostate Cancer (snapshot)

A 60-year-old man presents to his urologist for a 4-month history of pelvic pain. He denies fevers or chills and any recent sexual activity. His physical exam reveals a mildly tender prostate on digital rectal exam. His urinalysis is unremarkable. He is started on tamsulosin and ciprofloxacin.

Prostatitis (snapshot)

A 60-year-old man with a history of benign prostatic hyperplasia presents to his primary care physician for a 3-day period of fever, chills, and pain with urination. He was recently catheterized during an admission in the hospital. Physical exam reveals a tender and enlarged prostate on digital rectal exam. Urinalysis reveals pyuria and hematuria. He is started on empiric antibiotics.

Prostatitis(Snapshot)

A 13-year-old boy presents to the emergency department with a 1-hour history of nausea and vomiting. He says that these symptoms came on with right lower quadrant abdominal pain as well as pain in the right scrotum. Physical exam reveals an erythematous and swollen right scrotum. He also has tenderness to deep palpation of the right lower quadrant without guarding or rebound. Stroking of the thigh does not elicit any response. Which of the following mechanisms is most likely responsible for this patient's symptoms? A. Infection of the appendix B. Inflammation of the testis C. Sexually transmitted infection of epididymis D. Torsion of the broad ligament E. Twisting of processus vaginalis

This patient who presents with nausea, vomiting, and right scrotal pain with an absent cremasteric reflex most likely has testicular torsion, which is caused by twisting of the processus vaginalis. Testicular torsion is caused by twisting of the spermatic cord in the processus vaginalis. This twisting compromises blood flow to the testis and epididymis and the resulting ischemia leads to severe pain, nausea, and vomiting. Physical exam will reveal erythema, swelling, and tenderness to palpation of the affected testis. An important maneuver in this disease is the cremasteric reflex, mediated by nerve roots L1 and L2, which is elicited by stroking the inner thigh of the affected side. This should elicit contraction of the cremasteric muscle and elevation of the testis; however, in testicular torsion no response will be observed. This disease should be treated with manual detorsion and orchiopexy should be performed to prevent future torsion. Incorrect Answers: Answer 1: Infection of the appendix can cause right lower quadrant pain as well as nausea and vomiting; however, this disease would have rebound tenderness and would not explain the scrotal findings in this patient. Answer 2: Inflammation of the testis known as orchitis can also present with unilateral scrotal pain, erythema, and swelling; however, this disease would not present with an absent cremasteric reflex. This disease can be caused by a variety of organisms including mumps, Neisseria gonorrhoea, and Chlamydia trachomatis and should be treated with appropriate antibiotics if caused by bacteria. Answer 3: Sexually transmitted infection of epididymis classically presents with testicular pain and a tender epididymis; however, this disease is usually accompanied by increased urinary frequency and has an intact cremasteric reflex. This can be treated with ceftriaxone and doxycycline. Answer 4: Torsion of the broad ligament can cause ovarian torsion in female patients. This disease can present with abdominal pain, nausea, and vomiting; however, it would not be part of the differential in male patients. Bullet Summary: Testicular torsion presents with nausea, vomiting, and unilateral testicular pain with an absent cremasteric reflex.

A 48-year-old woman presents to the emergency department with a 1-day history of severe left back pain. She says that the pain is squeezing in character, occurs in waves, and radiates to her left groin. She has no past medical history, drinks socially, and does not smoke. Physical exam reveals left costovertebral tenderness, and urine sediment analysis reveals the finding shown in Figure A. Which of the following would most likely present with the same urine sediment findings as seen in this patient? A. Ethylene glycol abuse B. Gout C. Impaired amino acid reabsorption D. Infection E. Renal tubular acidosis

This patient who presents with severe colicky left flank pain that radiates to her left groin and has enveloped-shaped crystals on urine sediment analysis most likely has calcium oxalate nephrolithiasis, which can also be seen in ethylene glycol abuse. Nephrolithiasis is a disease where stones form in the kidneys or ureter and irritate the urinary tract. Many different types of stones exist and form under different circumstances. The most common of these stone types are calcium oxalate stones, which can occur in hypercalciuria, hypocitraturia, ethylene glycol poisoning, vitamin C abuse, and Crohn disease. These stones are radiopaque on both radiography and computed tomography scan. On urine sediment analysis, these stones will have an envelope shape. Figure/Illustration A show imaging of urine sediment with enveloped-shaped stones that are circled in red on the illustration. This stone shape is classically seen with calcium oxalate stones. Incorrect Answers: Answer 2: Gout is associated with the development of uric acid crystals; however, these stones would appear rhomboid in shape. Answer 3: Impaired amino acid reabsorption is associated with the development of cystine stones; however, these stones would appear hexagonal in shape. Answer 4: Infection is associated with the development of struvite stones; however, these stones would appear like a coffin lid in shape. Answer 5: Renal tubular acidosis is associated with the development of calcium phosphate stones; however, these stones would appear like a wedge-shaped prism. Bullet Summary: Calcium oxalate kidney stones will have an envelope or dumbbell shape on microscopy of urine sediment.

A 26-year-old man presents to his primary care physician with 3 days of dysuria. He reports to also noticing discharge arising from the penis and denies any scrotal or rectal pain. Medical history is significant for a prior sexually transmitted infection that was treated with oral azithromycin for one day. He says that he is sexually active with both men and women and infrequently uses condoms. On physical examination, there is purulent discharge at the urethral meatus. The patient receives a single intramuscular injection of ceftriaxone with a 1 day course of oral azithromycin.

Urethritis (snapshot)

A 22-year-old man presents to his primary care physician due to pain with urination and a burning sensation. The patient was in his usual state of health until 3 days prior to presentation. He reports to having unprotected sex for the past few weeks. On physical examination, there is purulent discharge at the urethral meatus. The patient receives a single intramuscular injection of ceftriaxone with a 1 day course of oral azithromycin.

Urethritis(SNAPSHOT)

A 48-year-old woman presents to the emergency department due to severe back pain. Her symptoms began approximately 3 hours ago and says the pain is in her left mid-back. She describes the pain as sharp and 9/10. The pain radiates to her left groin. On physical exam, there is left-sided costovertebral angle tenderness. A non-contrast computerized tomography (CT) scan of the abdomen demonstrates left-sided urolithiasis and hydronephrosis. She is started on a nonsteroidal antiinflammatory drug (NSAID) and intravenous normal saline.

(Nephrolithiasis resulting in hydronephrosis)

A 65-year-old previously healthy man presents to the primary care physician with the chief complaint of red colored urine over the past month. He states that he does not experience dysuria. On physical exam there is no costovertebral angle tenderness. With this presentation which is the most likely cause of this patient's hematuria? A) Bladder tumor B) Renal cell carcinoma C) Beeturia D) Urinary tract infection E) Urethral injury

A, Bladder Cancer In patients over the age of 40, a bladder tumor is the most likely cause of the development of painless hematuria. Transitional cell, squamous cell, and adenocarcinoma of the bladder are the 3 types of malignancies that can arise from the bladder. They often present in later stages with the patient having microscopic or gross hematuria. It is unlikely that a mass will be palpated, however abdominal pain and dysuria can be associated with a large mass. Smoking is associated with the development of transitional cell carcinoma. Treatment varies based on the extent of disease. Thaler and Wang review the evaluation of asymptomatic hematuria in adults. Routine screening in adults is not warranted because of the intermittent occurrence of this finding. Microscopic hematuria associated with anticoagulation therapy is a concerning sign and must be evaluated further. Jimbo discusses the management of hematuria from the perspective of a primary care physician. He places the importance on first determining whether the patient is experiencing true hematuria and then referring the patient to the correct specialty for further evaluation. Illustration A is an image of a bladder tumor on cystoscopy. Illustration B is a flow chart depicting the standard work up of a patient with hematuria. Incorrect Answers: Answer 2: Renal cell carcinoma is the second most common cause of painless hematuria in this age group. Answer 3: Consuming large quantities of beets can turn the urine red, but unlikely without a history of beet consumption. Answer 4: Pain is usually associated with a urinary tract infection and it is unlikely to result in hematuria. Answer 5: This patient has no history of a traumatic injury.

A 31-year-old man with no significant past medical history presents to his primary care provider complaining of right scrotal pain. He states that the pain began 2 days ago and has gradually worsened. He denies burning on urination. On further questioning, he reports two new sexual partners in the past 6 months. Vital signs T 99.8 F, HR 75 bpm, BP 125/80 mmHg, RR 12 rpm. Exam demonstrates a swollen right scrotum that is painful to the touch. Doppler ultrasonography of his right testicle is shown in Figure A. Urinalysis is positive for white blood cells. Of the following choices, which is the next best step? A) Ceftriaxone and doxycycline B) Levofloxacin C) Supportive care and MMR vaccine D) Referral to urologist E) CT abdomen and pelvis

A, Epididymitis The patient's presentation is consistent with acute epididymitis. Empiric treatment includes ceftriaxone plus doxycycline. Epididymitis is the most common cause of scrotal pain in adults males. It is typically caused by a retrograde bacterial infection, with N. gonorrheae and C. trachomatis being the most common pathogens in men 14-35 years of age. Empiric treatment is single dose intramuscular ceftriaxone and oral doxycycline for 10 days. In males younger than 14 or older than 35, urinary tract pathogens such as E. coli are most common and treatment options include levofloxacin for 10 days. Risk factors for the development of epididymitis include sexual activity, strenuous physical activity, bicycling, and prolonged periods of sitting. Symptoms usually include a gradual onset of scrotal pain with swelling, possible urinary tract symptoms, and fever. Trojian et al. discuss epididymitis and orchitis diagnosis and management, as well as the differential for scrotal pain. They emphasize that it is important to distinguish epididymitis from testicular torsion, which is often a surgical emergency. Doppler ultrasonography can be an important diagnostic tool in addition to history and physical in distinguishing these diagnoses. Besides antibiotics, symptom relief for epididymitis includes analgesics, scrotal elevation, limitation of activity, and use of cold packs. Tracy et al. discuss possible causes of noninfectious epididymitis, which include sarcoidosis, Behcet disease, Henoch-Schonlein purpura, and medication-induced (e.g. amiodarone). Diagnostic testing for infectious epididymitis also can include a gram stain of urethral exudate, PCR for chlamydia, and direct fluorescent antibody testing. Figure A shows an enlarged, thickened epididymis consistent with epididymitis. Illustration A is an ultrasound showing Doppler flow. Note that in addition to an enlarged epididymis, there is increased blood flow. This is in contrast to testicular torsion, where the testicle appears normal with decreased blood flow. Incorrect answers: Answer 2: Levofloxacin is recommended for complicated urinary tract infections; however, it is typically not a first line agent unless there is resistance to other preferred agents without plausible alternatives. Answer 3: This is the management of mumps if the patient has not yet received the vaccine. Answer 4: Referral to a urologist or a local ED may be indicated if testicular torsion is suspected. Answer 5: CT abdomen and pelvis is not indicated at this time.

A 53-year-old male presents to your office for abdominal discomfort. The patient states he first noticed pain on his right flank several months ago, and it has been gradually getting worse. For the past week, he has also noticed blood in his urine. Prior to this episode, he has been healthy and does not take any medications. The patient denies fever, chills, and dysuria. He has a 40 pack-year smoking history. Vital signs are T 37 C, HR 140/90 mmHg, HR 84/min, RR 14/min, O2 98%. Physical exam is unremarkable. CBC reveals a hemoglobin of 17 and hematocrit of 51%, and urinalysis is positive for red blood cells, negative for leukocytes. Which of the following is the most likely diagnosis? A) Renal cell carcinoma B) Polycystic kidney disease C) Abdominal aortic aneurysm D) Pyelonephritis E) Renal oncocytoma

A, Renal Cell Carcinoma Given the patient's presentation (flank pain, hematuria) and history of smoking, the most likely diagnosis is renal cell carcinoma. Renal cell carcinoma most commonly presents with a combination of hematuria, an abdominal mass, and flank discomfort, although many patients are asymptomatic and are diagnosed incidentally. Patients may also have an elevated hematocrit from elevated erythropoietin production. Risk factors for renal cell carcinoma include smoking, hypertension, obesity, acquired cystic kidney disease, and genetic factors such as von Hippel Lindau. Higgins and Fitzgerald discuss the differential diagnosis of renal and adrenal masses. They broadly categorize renal masses as cysts, tumors, or inflammatory lesions, and suggest ultrasound and/or abdominal CT with contrast as an initial diagnostic step. Cohen and McGovern review the diagnosis and management of renal cell carcinoma. They suggest that the standard of care has traditionally involved radical nephrectomy, although small tumors may be treated with partial nephrectomy. They also mention recent investigation into percutaneous ablative approaches, which may require further trials to determine its appropriate use in management. Incorrect answers: Answer 2: Polycystic kidney disease typically presents with bilateral complaints. Answer 3: Abdominal aortic aneurysm is less likely as the complaint appears to be renal in nature. Answer 4: The patient does not have evidence of infection. Answer 5: Renal oncocytoma is much less common than renal cell carcinoma, and is a benign tumor.

A 10-year-old boy presents to his pediatrician for a well child appointment. The patient has been doing well in school. He plays on a club basketball team and is also a member of the chess club. He has many friends and is very happy. His parents currently have no concerns for him. His temperature is 99.5°F (37.5°C), blood pressure is 127/68 mmHg, pulse is 85/min, respirations are 17/min, and oxygen saturation is 98% on room air. On physical exam, you note a tall, muscular young boy. He is in the 99th percentile for weight and height. Cardiopulmonary exam is within normal limits. The patient's abdomen is obese, non-tender, and non-distended. Neurological exam is grossly non-focal. Testicular exam is notable for a right-sided testicular mass. Musculoskeletal exam reveals a normal range of motion and 5/5 strength in his upper and lower extremities. Dermatologic exam reveals acne and facial hair on the patient's face. Which of the following is the most likely underlying diagnosis in this patient? A) Leydig cell tumor B) Normal development in the setting of obesity C) Pituitary adenoma D) Precocious puberty E) Sertoli cell tumor

A, Testicular Cancer This patient is presenting with a unilateral testicular mass and precocious puberty suggestive of a diagnosis of a Leydig cell tumor. Leydig cell tumors secrete hormones (in particular testosterone and estrogen) in a non-GnRH responsive pattern. In pediatric patients, the presentation can include precocious puberty, with findings such as increased height/weight and symptoms of virilization (deep voice, facial hair, and acne). In the setting of precocious puberty and a unilateral testicular mass, the diagnosis of a Leydig cell tumor is highly likely. Incorrect Answers: Answer 2: Normal development in the setting of obesity could be a likely diagnosis if this patient were only obese and experiencing precocious puberty, since obesity can induce precocious puberty. In this patient, the unilateral testicular mass suggests another diagnosis. Answer 3: A pituitary adenoma that secretes growth hormone could be possible given this patient's increased height; however, it does not correlate with his unilateral testicular mass. Answer 4: Precocious puberty is occurring in this patient; however, it is likely secondary to an underlying hormonally active Leydig cell tumor. Answer 5: Sertolli cell tumors could present with symptoms of precocious puberty and a unilateral testicular mass; however, they are epidemiologically less common than Leydig cell tumors for this clinical presentation. Bullet Summary: Leydig cell tumors present with a unilateral testicular mass and precocious puberty.

A 23-year-old man presents to the emergency department with testicular pain. His symptoms started 15 minutes ago and have not improved on the ride to the hospital. The patient's past medical history is non-contributory, and he is not currently taking any medications. His temperature is 98.5°F (36.9°C), blood pressure is 123/62 mmHg, pulse is 124/min, respirations are 18/min, and oxygen saturation is 98% on room air. Physical exam is notable for a non-tender abdomen. The patient's right testicle appears higher than his left and is held in a horizontal position. Stroking of the patient's medial thigh elicits no response. Which of the following is the best treatment for this patient? A) Bilateral surgical procedure B) Ceftriaxone C) Ciprofloxacin D) Manual detorsion E) Surgical debridement

A, Testicular Torsion This patient is presenting with sudden and severe testicular pain, a high riding and horizontal testicle, and an absent cremasteric reflex suggesting a diagnosis of testicular torsion, which is best managed with a bilateral orchiopexy. Testicular torsion occurs when the testicle twists around its blood supply causing ischemia and pain. If not treated rapidly, the ischemia can progress to necrosis leading to infection and infertility (secondary to exposed antigens from the damaged testicle triggering an immune response against the remaining testicle). The pathophysiology is related to abnormal fixation to the muscle and fascial coverings of the spermatic cord which allows the testicle to rotate freely on the spermatic cord within the tunica vaginalis. The optimal treatment of testicular torsion is rapid surgical treatment and bilateral fixation of both testicles to prevent future torsion. If surgical correction cannot be performed within 2 hours of patient presentation or if the patient's presentation is significantly delayed (such as if they waited to come in), manual detorsion can be performed. Incorrect Answers: Answer 2: Ceftriaxone is the appropriate treatment for epididymitis in a young man as it covers for Neisseria gonorrhoeae. It would present with tender testicles, a fever, and possible pain relief with elevation of the testicles. The testicle would not be high riding, and the cremasteric reflex would be normal in this condition. Answer 3: Ciprofloxacin is the appropriate treatment for epididymitis in an older man as it covers for Escherichia coli. It would present with tender testicles, a fever, and possible pain relief with elevation of the testicles. The testicle would not be high riding, and the cremasteric reflex would be normal in this condition. Answer 4: Manual detorsion is the optimal initial therapy for testicular torsion if surgical detorsion and bilateral fixation cannot be performed within a 2 hour time frame or if there has already been a significant delay in the patient's presentation, as every moment lost represents more testicular ischemia. Manual detorsion typically involves twisting the testicle from medial to lateral; however, roughly 1/3 of testicles twist in the opposite direction. Answer 5: Surgical debridement is the optimal treatment of Fournier gangrene (necrotizing fasciitis) which would present with a rapidly spreading infection with crepitus and progression of disease that can occur over minutes. Debridement and broad-spectrum antibiotics are a mandatory part of treatment. Bullet Summary: The best treatment of testicular torsion is surgical detorsion and bilateral fixation.

A 54-year-old woman presents to her gynecologist complaining of incontinence. She reports leakage of a small amount of urine when she coughs or laughs as well as occasionally when she is exercising. She denies any pain with urination. She underwent menopause 2 years ago and noted that this problem has increased in frequency since that time. Her history is significant only for three uncomplicated pregnancies with vaginal births. Urinalysis, post-void residual, and cystometrogram are conducted and all show normal results. The patient's vital signs are as follows: T 37.5 C, HR 80, BP 128/67, RR 12, and SpO2 99%. Physical examination is significant for pelvic organ prolapse on pelvic exam. Which of the following is a reasonable first step in the management of this patient's condition? A) Kegel exercises B) Imipramine C) Bethanechol D) Urethropexy E) Tension-free vaginal tape

A, Urinary Incontinence This patient is suffering from stress urinary incontinence. Noninvasive treatment options for stress incontinence include Kegel exercises to strengthen the pelvic floor. Stress incontinence (loss of urine with an increase in abdominal pressure, such as with coughing, laughing, and exercise) is caused by laxity of the pelvic floor muscles. This laxity allows for the proximal urethra to drop below the pelvic floor with increases in abdominal pressure. The change in angle between the urethra and bladder results in bladder pressure exceeding urethral pressure at times of strain and an ensuing involuntary loss of urine. Stress incontinence is associated with cystocele and decreased levels of estrogen, as seen in post-menopausal women. Beyond Kegel exercises, additional noninvasive treatment options for stress urinary incontinence include: bladder training/biofeedback, topical vaginal estrogen cream for postmenopausal patients, pessaries, vaginal cones, alpha-adrenergic agonists, duloxetine (SNRI), incontinence pads, weight loss, smoking cessation, fluid restriction, and avoidance of caffeinated beverages and alcohol (behavioral modification). Hersh et al. discuss the management of urinary incontinence in women. The approach to management should consist of progression from least invasive to more invasive treatment options, with surgery reserved for cases that have not responded to less invasive treatment options. Pelvic floor strengthening exercises are the first-line treatment for stress incontinence. Other noninvasive options include vaginal inserts, urethral plugs, or electrical and magnetic stimulation devices. Currently, no medications are approved by the USDA for treatment of stress incontinence. Luber reviews the prevalence of and risk factors for stress urinary incontinence. Stress urinary incontinence affects 4-35% of women. This high variability in the estimated prevalence of the disease is due to the lack of a standardized definition of the condition. With aging and obesity increasing and having causal relationships with stress urinary incontinence, the prevalence of stress urinary incontinence is expected to increase in the future. Illustration A shows how the change in the angle between the bladder and urethra that occurs with pelvic floor laxity at times of strain can lead to leakage or urine. Incorrect Answers: Answer 2: Anticholinergic agents such as imipramine have been shown to have minimal effect on the smooth muscle of the urethra; anticholinergics may be more effective for urge incontinence. Answer 3: Bethanechol is a cholinergic agonist that may be helpful in the treatment of outflow incontinence, not stress incontinence. Answer 4: Urethropexy is appropriate surgical treatment for stress incontinence; however, less invasive options should be tried first. Answer 5: Tension-free vaginal tape is a procedure that involves placement of mesh under the urethra - it is an invasive surgical procedure that should be pursued after failure of conservative management.

A 28-year-old man presents to a urologist upon referral from a fertility medicine specialist who evaluated the patient and his wife. The patient was told that he had a low sperm count. Otherwise, the patient endorses dull and low grade testicular pain that is chronic in nature and unchanged from his baseline. The patient's vitals are unremarkable. Examination and palpation of the right scrotum and testicle reveals soft palpable cords on the right side which are not seen on the left. Additionally, the examination reveals right testicular atrophy. When the patient lies supine, there is no change in the appearance or size of the scrotum. An ultrasound and color Doppler study of the patient's right testicle is shown in Figure A. Which of the following is the best next step in the management for this patient? A) CT abdomen and pelvis B) Embolization C) MRI pelvis D) Observation E) Surgical repair

A, Varicocele This patient's presentation (low sperm count and the 'bag of worms' finding on the testicular exam) is consistent with a varicocele. Right-sided varicocele, bilateral varicocele, and failure of a varicocele to disappear upon lying supine are signs suggestive of inferior vena cava (IVC) obstruction and warrant further investigation with a CT scan of the abdomen and pelvis. Varicocele is a dilation of the pampiniform venous plexus and spermatic veins. It is relatively common, occurring in approximately 10% of all men and is associated with infertility. Over 90% of varicoceles are left-sided, likely secondary to the comparable increased length of the left testicular vein and associated increased venous pressures. In contrast, the right testicular vein is shorter, making varicocele less likely due to lower venous pressures; therefore, when a right-sided varicocele is identified, the suspicion is raised for a force restricting return blood flow from the testicle to the IVC such as a compressive mass or malignancy. Potential causes may include an IVC thrombus or abdominal mass compressing the IVC. Prior to any further direct treatment of the varicocele, it is important to assess the patient with a CT abdomen and pelvis searching for a possible compressive mass. Figure A shows an ultrasound and color Doppler study of a patient with varicocele. Note the numerous anechoic tubes with bidirectional flow supporting the 'bag of worms' finding on physical exam. Incorrect Answers: Answers 2 & 5: Embolization and surgical repair are possible treatment options when indicated (such as cases where varicocele may be contributing to infertility); however, the signs in this patient's presentation that are suggestive of possible IVC obstruction warrant further evaluation with a CT abdomen. It would be inappropriate to directly treat the varicocele without addressing a possible underlying compressive mass. Answer 3: MRI of the pelvis does not have a role in evaluating varicocele. It may be indicated after a CT scan if there is any need for further characterization of soft tissues or a compressive mass but this is not a typical step in the workup. Answer 4: Observation is inappropriate as this patient needs to have a possible compressive mass ruled out and would need urology referral to treat his underlying infertility. Bullet Summary: Right-sided varicocele, bilateral varicocele, and failure of a varicocele to disappear upon lying supine warrant a CT scan to rule out inferior vena cava (IVC) obstruction.

A 67-year-old man presents to the emergency department with trouble urinating. The patient states that in general he has had difficulty urinating but recently, it has taken significant effort for him to initiate a urinary stream. He finds himself unable to completely void and states he has suprapubic tenderness as a result. These symptoms started suddenly 3 days ago. The patient has a history of benign prostatic hyperplasia, constipation, and diabetes mellitus. His current medications include finasteride, sodium docusate, and hydrochlorothiazide. He recently started taking phenylephrine for seasonal allergies. The patient's last bowel movement was 2 days ago. His temperature is 99.0°F (37.2°C), blood pressure is 167/98 mmHg, pulse is 90/min, respirations are 14/min, and oxygen saturation is 100% on room air. Physical exam is notable for suprapubic tenderness, and an ultrasound reveals 750 mL of fluid in the bladder. Which of the following is the most likely etiology of this patient's symptoms? A) Constipation B) Medication-induced symptoms C) Prostatic adenocarcinoma D) Urinary tract infection E) Worsening benign prostatic hypertrophy

B, Benign Prostatic Hyperplasia (BPH) This patient is presenting with sudden-onset urinary retention in the setting of benign prostatic hyperplasia (BPH) and starting an alpha-1 agonist (phenylephrine), suggesting his symptoms are secondary to this medication. Urinary retention typically presents in men with BPH and can present with straining to urinate, a painful and distended bladder, and incomplete voiding. Chronic treatment for BPH is finasteride, which is a 5-alpha-reductase inhibitor that can slow the progression of BPH. Acute treatments include tamsulosin, which is an alpha-1 antagonist which can aid in voiding. Patients who start medications with anticholinergic properties or alpha-1 agonist medications can worsen their urinary retention. Anticholinergics decrease cholinergic tone on the bladder which is required for urination. On the other hand, alpha-1 agonists increase the tone of the urinary tract sphincters making it more difficult to urinate. The treatment of medication-induced urinary retention is to stop the offending agent and to potentially start tamsulosin if symptoms persist. Incorrect Answers: Answer 1: Constipation is a potential etiology of urinary retention and could possibly be worsening this patient's symptoms of urinary retention. However, the patient's symptoms started 3 days ago, and his symptoms did not improve with a bowel movement 2 days ago. Though this patient's constipation should be better managed, it is not the most likely etiology of his urinary retention. Answer 3: Prostatic adenocarcinoma is unlikely to cause urinary retention when compared to benign prostatic hypertrophy. Prostatic adenocarcinoma typically presents with a rock hard and nodular prostate in contrast to a soft, symmetric, and enlarged prostate in BPH and is less likely to obstruct the urinary tract. Answer 4: Urinary tract infection would present with symptoms of dysuria including urinary urgency, burning, and increased frequency rather than urinary retention. Answer 5: Worsening benign prostatic hyperplasia is a possible contributing etiology to this patient's urinary retention; however, it would not cause a sudden onset of urinary retention as was seen in this patient. Rather, it would be a gradual worsening of urinary retention. Bullet Summary: Anticholinergic medications and alpha-1 agonists can cause or exacerbate urinary retention.

A 73-year-old man presents to his primary care physician complaining of increased urinary frequency, nocturia, and incomplete emptying after void. He is otherwise healthy, with no active medical problems. On examination, a large, symmetric, firm, smooth prostate is palpated, but otherwise the exam is normal. Which of the following is a potential complication of the patient's present condition? A) Prostate cancer B) Hydronephrosis C) Bladder cancer D) Prostatitis E) Renal cancer

B, Benign Prostatic Hyperplasia (BPH) This patient presents with signs and symptoms consistent with benign prostatic hypertrophy (BPH); this condition places one at increased risk for hydronephrosis. BPH is a common condition seen in older men. It typically presents with any number of lower urinary tract symptoms ("LUTS"), including: increased frequency, nocturia, hesitancy, urgency, and weak stream. Enlarged prostate on examination can confirm the diagnosis. Over time, the retrograde pressure caused by BPH can cause hydronephrosis and even renal failure; this is also associated with bladder diverticula. Patients with BPH are at increased risk for developing urinary tract infections, but not prostate cancer. Incorrect Answers: Answers 1, 3, 4: All of these are not associated with BPH, whereas hydronephrosis is. Answer 5: Although bladder stones can be associated with BPH, nephrolithiasis is not.

A 6-month-old male presents with a painless, enlarged left scrotum. After examining the patient, you suspect this enlargement is secondary to serous fluid entering and accumulating in the scrotum through a patent processus vaginalis. Which of the following would be the most useful next step in confirming the diagnosis of this patient's condition? A) Measurement of AFP and hCG levels B) Transillumination test followed by scrotal ultrasound C) Evaluation of cremasteric reflex on physical exam D) Measurement of serum testosterone levels E) Standard urinalysis

B, Hydrocele This patient's presentation is consistent with a left-sided hydrocele. The diagnosis of hydrocele is confirmed through a positive illumination test and a scrotal ultrasound, which can differentiate solid masses from fluid. Hydrocele often occurs due to failed obliteration of the processus vaginalis, the communication between the tunica vaginalis and the peritoneum. Fluid, such as serous fluid, blood (hematocele), or sperm (spermatocele), accumulates in the tunica vaginalis of the scrotum, resulting in the enlarged appearance. Hydrocele is often associated with indirect inguinal hernia. Surgery is indicated to close the patent processus vaginalis. Galejs discusses the diagnosis and management of scrotal abnormalities. In evaluating such patients, physical examination should include inspection and palpation of the abdomen, testes, epididymis, scrotum, and inguinal region. Urinalysis should routinely be performed; however, scrotal imaging such as ultrasound should only be pursued when the diagnosis remains unclear after initial evaluation. Lao et al. address the management of hydrocele in very young patients, concluding that hydroceles are relatively common in infants and often resolve without operative intervention. As a general rule, hydroceles should be expected to resolve by 1 year of age. After this time period, surgery should be pursued to close a persistent patent processus vaginalis and thereby decrease the risk of future complications such as inguinal hernias. Illustration A depicts a positive transillumination test demonstrating the presence of hydrocele. Illustration B shows the appearance of hydrocele on ultrasound. Incorrect Answers: Answer 1: These tumor markers would be useful in the diagnosis of a testicular tumor; generally one or both would be expected to be increased, depending on the type of tumor present. Answer 3: This finding is consistent with testicular torsion, which would present with a sudden onset of acute testicular pain. Answer 4: Serum testosterone levels would not be expected to be disturbed by a hydrocele. Answer 5: No abnormalities would be anticipated on urinalysis, therefore, it is unlikely to be helpful.

A 64-year-old man presents to the emergency department with acute onset of chest pain. He says the pain is substernal and radiates to his left arm. He has a history of hypertension, diabetes mellitus, erectile dysfunction, benign prostate hyperplasia, and panic disorder. He takes aspirin, lisinopril, metformin, sildenafil, prazosin, and citalopram. An electrocardiogram shows new ST-elevations in the lateral leads. He undergoes catherization, which reveals a complete blockage of the left circumflex artery. A stent is placed, and the patient is discharged with clopidogrel and isosorbide mononitrate. Five days later the patient presents to the emergency department complaining of fainting spells. The patient's temperature is 97°F (37.2°C), blood pressure is 89/53 mmHg, and pulse is 90/min. Physical examination is unremarkable. An electrocardiogram reveals lateral Q waves without ST or T wave abnormalities. Which of the following is the most likely cause of the patient's presentation? A) Fibrinous pericarditis B) Medication interaction C) Myocardial wall rupture D) Papillary muscle rupture E) Stent thrombosis

B, Impotence / Erectile Dysfunction The patient presents with fainting spells and hypotension after being started on isosorbide mononitrate while he is already taking sildenafil and prazosin. As sildenafil's use is contraindicated with a nitrate, this is suspicious for a medication interaction. Sildenafil is a phosphodiesterase-5 (PDE-5) inhibitor that acts as a vasodilator to lower blood pressure and causes smooth muscle relaxation. It is used to treat pulmonary hypertension and erectile dysfunction. PDE-5 inhibitors work similarly to nitrates by using a cyclic GMP mechanism to prevent the degradation of nitric oxide. Therefore, nitrates can potentiate the effects of PDE-5 inhibitors, and their simultaneous use is contraindicated. Patients who use both PDE-5 inhibitors and nitrates are at risk for severe hypotension and syncope, especially if they also taking other antihypertensive medications. Incorrect Answers: Answer 1: Fibrinous pericarditis occurs 1-3 days post-myocardial infarction. Patients present with chest pain and a friction rub. Answer 3: Myocardial wall rupture occurs 3-14 days post-myocardial infarction. Patients present with symptoms of chest pain, syncope, jugular vein distension, signs of cardiac tamponade, and sudden death. Answer 4: Papillary muscle rupture occurs 3-14 days post-myocardial infarction. Patients present with signs of mitral regurgitation, such as a holosystolic murmur heard best at the apex, a S3 heart sound, and lung crackles. Answer 5: Stent thrombosis often results in symptoms of angina. An electrocardiogram would show ST-elevations in leads similar to those seen with the patient's initial myocardial infarction. Bullet Summary: Sildenafil is a PDE-5 inhibitor, and its use with nitrates is contraindicated.

A 47-year-old female presents to her primary physician for follow up after an ED visit for nephrolithiasis 1 week prior. CT scan in the ED revealed a 4-mm stone occluding the right ureter. She was able to pass the stone without assistance over the course of 5 hours. Pathology report on the stone states it was composed of calcium oxalate. She says that the pain of passing the stone was excruciating and asks how she can prevent kidney stones in the future. Which of the following interventions is most likely to prevent renal calculi formation in the future for this patient? A) Decreasing calcium intake B) Decreasing sodium intake C) Increasing consumption of nuts and seeds D) Loop diuretic pharmacotherapy E) Urine acidification with cranberry juice

B, Nephrolithiasis Decreasing sodium intake is the only intervention that will reduce the risk of calcium oxalate stone formation. Calcium oxalate stones are the most common type of renal stone. Prevention of calcium oxalate stones focuses on reducing the concentration of calcium and oxalate within the renal collecting system, thereby preventing precipitation and stone formation. Calcium resorption occurs primarily in the proximal tubule and is dependent on sodium concentration. Reduction in dietary sodium allows more calcium to be resorbed in the proximal tubule, thereby decreasing stone formation. Other methods of preventing calcium oxalate stone formation includes increasing fluid intake and adding a thiazide diuretic if lifestyle modifications are not effective. Incorrect Answers: Answer 1: Decreasing calcium intake is only recommended if consumption is excessive. It is generally recommended to consume normal amounts of calcium in these patients. Answer 3: Increasing consumption of nuts and seeds is unlikely to prevent future calcium stone formation. Additionally, certain nuts contain high levels of oxalate. Answer 4: Loop diuretics are not indicated for renal stone prophylaxis. Thiazide diuretics however have been shown to reduce stone formation. Answer 5: Cranberry juice, while capable of acidifying the urine has been shown to increase urine oxalate and calcium levels. It has not been shown to reduce the risk of stone formation. Bullet Summary: Decreasing dietary sodium, increasing fluid intake, and thiazide diuretics are interventions that can be used to reduce the risk of calcium oxalate stone formation.

A 64-year-old man presents to the clinic complaining of low back pain. He was diagnosed with benign prostatic hypertrophy several years ago, which worsened last year. A hard nodule was felt on a digital rectal exam, which led to a prostate biopsy that revealed high-grade adenocarcinoma. A bone scan at that time showed small, focal abnormalities in the lumbar spine. He subsequently underwent radiation therapy to his prostate and pelvic lymph nodes. He was then placed on leuprolide and an anti-androgen. He was doing well until 2 months ago when he developed low back pain, which has worsened since then. He reports that the pain is 8/10 and nearly constant. He states that the pain not only affects his ability to fall asleep but also wakes him up at night. The pain is mildly alleviated by ibuprofen. In addition to prostate cancer, his medical history is significant for hypertension, hyperlipidemia, recurrent nephrolithiasis, and gastroesophageal reflux disease. He takes aspirin, lisinopril, rosuvastatin, and pantoprazole. Upon physical examination, there is midline tenderness within the lumbar region. Magnetic resonance imaging of the spine is obtained, which shows bony metastasis to the L2 and L3 spine without evidence of fracture or spinal cord compression. His prostate-specific antigen is 17.5 ng/mL. Which of the following is the best next step in management? A) Calcium and vitamin D supplementation B) External radiation therapy C) Flutamide D) Kyphoplasty E) Zoledronic acid

B, Prostate Cancer The patient is presenting with prostate cancer with symptomatic metastasis to the bone. The patient is already on a maintenance gonadotropin-releasing hormone (GnRH) agonist (leuprolide), so the next best step in management would be external radiation therapy. Osteoblastic lesions in the bone are the most common site of metastasis for prostate cancer. Frequently, these lesions cause pain and even functional impairment. If the patient is already on a GnRH agonist, such as leuprolide, the treatment of choice for bone pain is external beam radiation therapy. Relief of bone pain following external beam radiation is usually rapid, even in those with poor prognosis. Supportive pain medications can also be used in conjunction, and the literature suggests dexamethasone may reduce pain flares for patients receiving external beam radiation therapy. Incorrect Answers: Answer 1: Calcium and vitamin D supplementation should be used prior to initiating an osteoclast inhibitor if calcium or vitamin D levels need to be corrected. There are contraindications, including recurrent renal stones, which this patient has. Additionally, calcium and vitamin D supplementation themselves will not treat the patient's bone pain. Answer 3: Flutamide is an anti-androgen. It is the initial best step when beginning GnRH analogs to prevent the first initial dose having a pulsatile effect and worsening metastasis. This patient was already placed on an anti-androgen when the GnRH analog was started, and he is now on maintenance therapy. Answer 4: Kyphoplasty can be used for bone lesions in patients with metastatic prostate cancer, but it is usually reserved for patients with pathologic fractures or signs of spinal cord compression. Answer 5: Zoledronic acid is a bisphosphonate. Literature shows that bisphosphonates may be an alternative to external beam radiation therapy if there are contraindications to radiation or if the patient is refractory to radiation. However, it has not been approved for this indication. Bisphosphonate are more often used to prevent or delay bone-related events. Bullet Summary: In a patient with symptomatic bone metastasis who is currently on maintenance GnRH therapy, the next best step in management is external radiation therapy to shrink the metastatic bone lesions.

A 30-year-old male presents with a testicular mass of unknown duration. The patient states he first noticed something unusual with his right testicle two weeks ago, but states he did not think it was urgent because it was not painful and believed it would resolve on its own. It has not changed since he first noticed the mass, and the patient still denies pain. On exam, the patient's right testicle is non-tender, and a firm mass is felt. There is a negative transillumination test, and the mass is non-reducible. Which of the following is the best next step in management? A) Needle biopsy B) Testicular ultrasound C) MRI abdomen and pelvis D) CT abdomen and pelvis E) Send labs

B, Testicular Cancer The next step in evaluation of scrotal masses after the physical exam and transillumination test is testicular ultrasonography to help determine if the mass is extra-testicular or intra-testicular. The differential for scrotal masses includes torsion, orchitis, epididymitis, hematocele, hydrocele, varicocele, hernia, and testicular cancer. Testicular cancer is the most common malignancy in males of age 15-35, the vast majority of which are germ cell tumors. It typically presents as a non-acute painless testicular mass, although some patients do report discomfort. Other manifestations may include gynecomastia, paraneoplastic hyperthyroidism, and paraneoplastic limbic encephalitis. Once diagnosed, the first step is radical inguinal orchiectomy with possible retroperitoneal lymph node dissection to evaluate for metastasis. Following surgery and staging, chemotherapy, radiation therapy, or active surveillance are all viable options. Incorrect Answers: Answer 1: Needle biopsy is not indicated in the workup of suspected testicular cancer. Answer 3 and 4: Imaging of the abdomen and pelvis may be necessary once the diagnosis has been established to evaluate for metastasis. Answer 5: Beta-HCG, AFP, and LDH are some tumor markers seen in patients with testicular cancer and should be ordered once the ultrasound reveals an intra-testicular mass.

A 49-year-old woman comes to the office complaining of 2 weeks of urinary incontinence. She says she first noticed some light, urinary dribbling that would increase with sneezing or coughing. This dribble soon worsened, soaking through a pad every 3 hours. She denies any fevers, chills, abdominal pain, hematuria, dysuria, abnormal vaginal discharge, or increased urinary frequency. The patient had a bilateral tubal ligation 3 weeks ago. Her last menstrual period was 2 weeks ago. Her menses are regular and last 5 days. She has had 3 pregnancies that each resulted in uncomplicated, term vaginal deliveries. Her last pregnancy was 2 years ago. The patient has hypothyroidism and takes daily levothyroxine. She denies tobacco, alcohol, or illicit drug use. She has no history of sexually transmitted diseases. She is sexually active with her husband of 25 years. Her BMI is 26 kg/m^2. On physical examination, the abdomen is soft, nondistended, and nontender without palpable masses or hepatosplenomegaly. Rectal tone is normal. The uterus is anteverted, mobile, and nontender. There are no adnexal masses. Urine is seen pooling in the vaginal vault. Urinalysis is unremarkable. Which of the following is next best step in diagnosis? A) Cystoscopy B) Methylene blue instillation into the bladder C) Post-void residual volume D) Transvaginal ultrasound E) Q-tip test

B, Urinary Incontinence The patient is presenting with continuous, painless urinary leakage and pooling of fluid in the vagina following bilateral tubal ligation, which is consistent with a vesicovaginal fistula. Methylene blue instilled into the bladder can confirm the diagnosis. A vesicovaginal fistula may present following pelvic or abdominal surgery, prolonged labor, or pelvic irradiation. Patients complain of painless, continuous leakage of urine from the vagina. There is an increased risk for urinary tract infections (UTIs) and cystitis due to vaginal flora. On exam, the key diagnostic finding is fluid pooling in the vagina. To confirm the diagnosis, methylene blue can be instilled into the bladder. If a tampon placed in the vagina turns blue, the test is positive. Incorrect Answers: Answer 1: Cystoscopy is indicated in patients with persistent hematuria to assess for malignancy or bladder pain. Answer 3: Post-void residual volume can be used to evaluate for overflow incontinence, which occurs as a result of neurogenic bladder such as caused by diabetic neuropathy. Patients present with constant urinary dribbling due to incomplete bladder emptying. Risk factors for neurogenic bladder and evidence of an over-distended bladder are not present in this patient. Answer 4: Transvaginal ultrasound may confirm the diagnosis of a urethral diverticulum. Patients present with post-void dribbling, dysuria, dyspareunia, and an anterior vaginal wall mass. Answer 5: Q-tip test for urethral hypermobility may help diagnosis stress urinary incontinence. Patients present with leaking only with increase in intraabdominal pressure such as with cough, lifting, sneezing, or Valsalva. Patients would not have constant dribbling/leakage nor pooling of fluid in the vagina. Bullet Summary: A vesicovaginal fistula can be confirmed with methylene blue dye instilled into the bladder.

A 31-year-old G2P2 woman gave birth to her second child via vaginal delivery 8 hours ago. The patient received an epidural for pain control and delivery was uncomplicated. The patient's pain is currently well-controlled, and she is resting comfortably. The patient's mother comes to sit on the bed next to the patient and notes that the patient's bed is wet. When you examine the patient, you note some suprapubic tenderness with abdominal exam. A bladder scan reveals a distended bladder. The patient has not voided since delivery. What is the best initial step in management for this patient? A) Administer oxybutynin B) Begin intermittent urinary catheterization C) Obtain urinalysis and urine culture D) Obtain urodynamic testing E) Place an indwelling Foley catheter

B, Urinary Incontinence This patient has a lack of spontaneous micturition within 6 hours of vaginal delivery, as well as bladder distention and overflow incontinence, and therefore likely has postpartum urinary retention. The best next step in management of postpartum urinary retention is intermittent urinary catheterization. Postpartum urinary retention (PUR) occurs due to pudendal nerve injury during delivery. Women are at increased risk for PUR if it is their first delivery or if they receive epidural anesthesia, instrument-assisted delivery, or episiotomy during delivery. PUR is commonly defined as absence of spontaneous micturition within six hours of vaginal delivery or within six hours of removal of an indwelling catheter after cesarean delivery. Patients may be asymptomatic or experience urinary overflow incontinence, small voiding volumes, urinary frequency or urgency, bladder pain or discomfort, a sense of incomplete emptying, or an absence of the sensation to void. Bladder distension may be palpated on exam or visualized by ultrasound. Intermittent urinary catheterization is the management of choice for postpartum urinary retention. Ideally, catheterization is performed by the patient herself. Catheterization should be performed every 4-6 hours or whenever the patient has the urge to void but is unable to do so. If the patient voids a small volume, then she should perform self-catheterization afterwards to determine the residual volume. Catheterization can be discontinued when the residual urine volume is < 150 mL, and the patient is no longer experiencing significant symptoms of difficulty voiding. For most patients, urinary retention resolves within 1 week after delivery. Incorrect Answers: Answer 1: Oxybutynin is an antimuscarinic agent used for treatment of urinary urgency, incontinence, and overactive bladder in women. Medications, however, have proven ineffective at treating postpartum urinary retention; therefore, oxybutynin should not be used for this patient. Answer 3: Although some symptoms of postpartum urinary retention may resemble a urinary tract infection, such as bladder pain and increased urinary urgency or frequency, this patient's lack of spontaneous voiding within 6 hours after vaginal delivery, her distended bladder seen on bladder scan, and her symptoms of overflow incontinence are all consistent with postpartum urinary retention and should be managed with urinary catheterization. Answer 4: Urodynamic testing involves a set of tests that study the function of the urinary tract. It may be used to evaluate patients who are experiencing urinary urgency or incontinence. Postpartum urinary retention generally resolves on its own within 1 week and does not require urodynamic testing as initial management. If symptoms persist for a longer period, then further evaluation of the urinary tract may be indicated. Answer 5: Intermittent catheterization is the preferred management for postpartum urinary retention over indwelling Foley catheters, since indwelling catheters have a higher complication rate than intermittent catheterization, including higher risk of subsequent urinary tract infection. Bullet Summary: Postpartum urinary retention is a self-resolving condition that often presents after regional analgesia or cesarean section with inability to void, and is best managed with intermittent urinary catheterization.

A 44-year-old man presents to his primary care physician with several months of worsening difficulty attaining an erection. He reports that his libido has also declined. He denies feeling depressed or having relationship difficulties with his wife. The patient has a past medical history of gastroesophageal reflux disorder (GERD) and asthma, for which he sometimes uses an albuterol inhaler. He had an appendectomy at age 16. He has a family history of type II diabetes mellitus in his father and ovarian cancer in his mother. At this visit, the patient's temperature is 98.8°F (37.1°C), blood pressure is 139/83 mmHg, pulse is 80/min, and respirations are 13/min. Cardiovascular and pulmonary exams are within normal limits, and examination of the chest is shown in Figure A. Palpation of the testicles reveals a nontender, nonmobile one-centimeter nodule on the right testicle. The patient is sent for an ultrasound, and an image of the right testicle is shown in Figure B. Which of the following lab values would be expected in this patient? A) Elevated lactate dehydrogenase B) Decreased luteinizing hormone C) Elevated alpha-fetoprotein D) Decreased testosterone E) Increased human chorionic gonadotropin

B,Testicular Cancer This patient is a middle-aged man with decreased libido and difficulty attaining erection, with a testicular nodule, most consistent with a Leydig cell tumor. This tumor classically produces testosterone, which is then aromatized into estrogen, resulting in suppression of luteinizing hormone. Excess estrogen in a male can arise from an estrogen-producing tumor, or from a testosterone-producing tumor with peripheral aromatization to estrogen. Other symptoms of excess estrogen may include infertility, testicular atrophy, and feminization of hair patterns. In a patient with a testicular mass that is palpated on exam and confirmed on ultrasound, the most likely diagnosis is a Leydig cell tumor, a sex cord stromal tumor. Because of the excess estrogen, luteinizing hormone and follicle stimulating hormone will be low as a result of negative feedback. Figure A shows bilateral gynecomastia. Figure B shows an ultrasound of the right testicle with a well circumscribed, heterogeneously hypoechoic mass. Incorrect Answers: Answer 1: Elevated lactate dehydrogenase (LDH) may be found in seminomas, which may present as testicular masses or rarely occur in the mediastinum. Endocrine symptoms are typically not seen. High LDH may also be a result of excess tissue breakdown (e.g., tumor lysis syndrome) or hemolysis. Answer 3: Elevated alpha-fetoprotein (AFP) can be seen in nonseminomatous germ cell tumors, such as yolk sac tumors or embryonal carcinoma. These tumors generally do not produce testosterone or estrogen and thus would not cause this patient's symptoms. This lab finding may also be seen in hepatocellular carcinoma or chronic liver disease. Answer 4: Decreased testosterone from hypogonadism may present with symptoms similar to those seen in this patient. However, the presence of a testicular mass suggests that there is an underlying tumor producing the endocrine symptoms. Low testosterone may be due to medications (e.g., chronic steroid use), chronic medical conditions (e.g., diabetes), or pituitary lesions. Answer 5: Increased human chorionic gonadotropin (hCG) may be produced by embryonal carcinoma or choriocarcinoma. Neither of these would result in symptoms of estrogen excess as seen in this patient. Bullet Summary: Negative feedback from increased testosterone results in low luteinizing and follicle stimulating hormones in Leydig cell tumors.

A 70-year-old man presents to his physician with difficulty initiating a stream and post-void dribbling. He also reports having increased urinary urgency and a weak urinary stream. Medical history is significant for hypertension and is not on medication. On digital rectal exam, his prostate is enlarged, non-tender, firm, and smooth. Urinalysis is unremarkable and prostate-specific antigen is elevated. He is started on tamsulosin to improve his lower urinary tract symptoms.

Benign Prostatic Hyperplasia (BPH) (snapshot)

A 65-year-old man presents to his primary care physician after noticing blood in his urine. He reports that his urinary stream is normal but notices blood at the end of urination. He denies any abdominal pain, dysuria, or urinary frequency or urgency. Medical history is unremarkable. He smokes 1 pack of cigarettes daily for the past 35 years. Physical examination is unremarkable. Urinalysis demonstrates a large red blood cell count with normal red blood cell morphology. He eventually undergoes cystoscopy, which demonstrates a protruding mass from the bladder wall.

Bladder Cancer (snapshot)

A 55-year-old man presents to the emergency department with painless hematuria. He denies any trauma to the abdomen or pelvis, as well as any signs of fevers or chills. He denies any dysuria, urinary urgency, or frequency. He has not started any new diets. He smokes 1 pack of cigarettes daily for the past 35 years. Physical examination is not notable for any abdominal masses or tenderness to palpation. Urinalysis demonstrates a large red blood cell count with normal red blood cell morphology. He eventually undergoes cystoscopy, which demonstrates a protruding mass from the bladder wall.

BLADDER CANCER(SNAPSHOT)

A 52-year-old man presents to his primary care physician with nocturia and increased day-time urinary frequency. He notices that he needs to strain when beginning to void and tends to dribble after he finishes urinating. Medical history is significant for hypertension treated with hydrochlorothiazide and type 2 diabetes mellitus treated with metformin. On digital rectal exam, there is a non-tender, smooth, elastic, and firm enlarged prostate. A urinalysis and prostate-specific antigen level is ordered.

BPH Snapshot

A 55-year-old male comes to the emergency department complaining of blood in his urine and severe back pain. He reports that the pain is left-sided, colicky, and began acutely this morning. The pain occasional radiates to his left groin. He also reports increased urinary frequency and an episode of blood-tinged urine. According to the patient, this has happened multiple times before. His medical history is also notable for hypertension, type II diabetes mellitus, and obstructive sleep apnea. His current medications include aspirin, metformin, and lisinopril. The patient's temperature is 100.4°F (38°C), blood pressure is 148/85 mmHg, pulse is 71/min, and respirations are 14/min with an oxygen saturation of 98% on room air. Physical examination is significant for left-sided costovertebral angle tenderness. A urine dipstick is heme-positive. A urinalysis is pending. Labs are drawn, as shown below: Serum: Na+: 144 mEq/L Cl-: 95 mEq/L K+: 4.3 mEq/L HCO3-: 23 mEq/L Urea nitrogen: 18 mg/dL Glucose: 142 mg/dL Creatinine: 1.0 mg/dL Calcium: 7.8 mg/dL A radiograph is obtained, as shown in Figure A. In addition to increasing fluid intake, which of the following can help the patient prevent future episodes of the most likely diagnosis? A) Allopurinol B) Furosemide C) Hydrochlorothiazide D) Increase protein intake E) Urine alkalinization

C, Nephrolithiasis The patient is presenting with radiopaque kidney stones and mild hypocalcemia, suggesting calcium-containing nephrolithaisis. Thiazide diuretics, such as hydrochlorothiazide, can be used to help prevent future episodes of calcium kidney stones. Nephrolithasis presents as colicky flank pain (that may radiate to the groin or abdomen), dysuria, hematuria, urinary urgency and frequency, and costovertebral angle tenderness. Calcium-containing kidney stones may be caused by hypercalciuria, hypocitraturia, ethylene glycol, vitamin C abuse, Crohn disease, and urine alkalinization. On radiography and computerized tomography (CT), calcium-containing stones appear radiopaque. On urinalysis, crystals may be envelope-, dumbbell-, or wedge-shaped. Measures to prevent future episodes include increasing fluid consumption, adhering to a low-sodium diet, decreasing protein intake, taking citrate and thiazide diuretics. Thiazide diuretics decreases urinary calcium, thus decreasing stone formation. Figure A is a radiograph showing radiopaque kidney stones, which is characteristic of calcium-containing nephrolithiasis. Incorrect Answers: Answer 1: Allopurinol may be used as part of the management for uric acid-containing kidney stones. Causes of uric acid stones include states of increased cell turnover and gout that result in hyperuricemia and states of acidic urine. On radiography, uric acid stones are radiolucent, and may be minimally visible on CT. Answer 2: Furosemide is a loop diuretic that increases urinary calcium, which may precipitate calcium kidney stones. Answer 4: Increasing protein intake may increase the risk of kidney stones by increasing renal excretion. Answer 5: Urine alkalinization is known to precipitate calcium phosphate stones. It can be used to prevent uric acid stones, which precipitate in acidic urine. Bullet Summary: Calcium-containing kidney stones are radiopaque and can be prevented with increased fluid consumption, thiazide diuretic administration, and low salt and protein intake.

A 29-year-old G1P0 at 23 weeks of gestation presents to the ED for left flank pain migrating to the groin. The pain is sharp, causing her to have nausea and vomiting. She also endorses urinary frequency, but denies vaginal discharge. There have been no complications in her pregnancy thus far. Her abdominal exam is remarkable for left lower quadrant tenderness to palpation with pain radiating to the left groin, but no guarding. She also has tenderness to palpation of the left flank. Blood is visible on inspection of the perineal area. Urinalysis: Urine Color: Yellow pH: 7.1 Specific gravity: 1.010 Blood: 3+ Bilirubin: Negative Glucose: Negative Ketones: Negative Protein: Negative Nitrite: Negative Leukocyte esterase: Negative Red blood cells: 291 cells/ul White blood cells: 75 cells/ul Which of the following is the next best step in management? A) Renal radiograph B) Intravenous pyelogram C) Renal ultrasound D) Noncontrast CT scan of abdomen and pelvis E) Exploratory laparoscopy

C, Nephrolithiasis This patient is presenting with flank pain radiating to the groin and hematuria suggesting a diagnosis of nephrolithiasis (kidney stone). Since the patient is pregnant, the most appropriate next step is to obtain an abdominal ultrasound. Symptoms of nephrolithiasis include colicky pain (depending on where the stone is located) radiating to the groin, dysuria, urgency, hematuria, and nausea/vomiting. Leukocytosis may be present without evidence of infection. In a nonpregnant patient, a CT scan without contrast is indicated to detect renal stones and rule out other etiologies of abdominal pain with high sensitivity and specificity. However, in a pregnant patient, CT imaging carries the risk of radiation exposure to the fetus. To avoid this, ultrasound is the best initial imaging in a pregnant woman. Teratogenic effects of drugs and imaging are commonly assessed. With the advent of ultrasound, diagnosis of intraabdominal infections, aneurysms, and other processes have been made much safer during pregnancy. Always consider ultrasound first among management options if imaging is needed. Incorrect Answers: Answer 1: A plain film will only be able to detect large radiopaque stones but may miss radiolucent stones such as uric acid or smaller size stones. Answer 2: An intravegnous pyelogram has higher sensitivity and specificity for detecting stones than a plain film and can show the degree of obstruction. However, this modality has fallen out of favor due to risk of contrast reaction and radiation exposure. Answer 4: A helical CT scan would normally be indicated since it has high sensitivity and specificity. However, this would expose the fetus to a large amount of radiation. Answer 5: Exploratory laporoscopy is an invasive procedure and some type of imaging would be useful for guidance before subjecting the patient to unnecessary risk and complication, especially in the setting of pregnancy. Bullet Summary: If nephrolithiasis occurs during pregnancy, an abomindal ultrasound should be used to evaluate the patient.

A 68-year-old man presents to his primary care physician with complaints of intermittent dysuria, pain with ejaculation, mild lower abdominal pain, and difficulty voiding for the last four months. There is no weight loss or change in stools. He has no known family history of cancer. His past medical history is notable for irritable bowel syndrome and hypertension. On examination, he is well-appearing but mildly uncomfortable. There are no abdominal or rectal masses appreciated; the prostate is mildly tender to palpation, but with normal size, texture, and contour. Urinalysis reveals trace leukocyte esterase and negative nitrite, negative blood, and no bacteria on microscopy. Which of the following is the most appropriate treatment? A) Tamsulosin B) Ciprofloxacin C) Tamsulosin and ciprofloxacin D) Finasteride E) Duloxetine

C, Prostatitis This patient presents with a somewhat prolonged prodrome of symptoms suggestive of chronic prostatitis. The mainstay of treatment for this condition is alpha blockers and antibiotics. Chronic prostatitis is a syndrome consisting of lower urinary tract symptoms and pelvic pain. It is a distinct entity from chronic bacterial prostatitis. Prominent symptoms include pain that can involve the lower abdomen, penis, testicles, or prostate area. There is often concurrent bladder irritation or obstruction that leads to voiding difficulty. Urinalysis can reveal pyuria, but not always. It can be associated with irritable bowel syndrome and fibromyalgia. Front-line treatment for chronic prostatitis includes alpha blockers and fluoroquinolone antibiotics. The clinical syndrome described can overlap with other conditions such as benign prostatic hypertrophy (BPH), prostate cancer, and chronic bacterial prostatitis. BPH typically has a predominance of voiding symptoms rather than dysuria. Prostate cancer could potentially present in this way; however, none of the given answer choices would be appropriate for the treatment of prostate cancer. Finally, chronic bacterial prostatitis is a similar condition, but the question stem should emphasize bacteruria or even prostatic fluid bacterial positivity, which is the best diagnostic method for this. After diagnosing chronic prostatitis, one must know that the initial therapy is BOTH alpha blockers and antibiotics, and that finasteride and antidepressants can be adjunctive therapies. Incorrect Answers: Answers 1 and 2: As stated above, the treatment of chronic prostatitis consists of combination therapy with alpha blockers and antibiotics. Antibiotics alone would be indicated for a urinary tract infection or chronic bacterial prostatitis. Alpha blockers alone are indicated for uncomplicated BPH. Answers 4 and 5: Although finasteride (a 5-alpha reductase inhibitor) and duloxetine (a serotonin-norepinephrine reuptake inhibitor) can be used as adjunctive agents for patients refractory to the initial therapies, they should not be used first-line. Prostatitis is a common condition in general medical practice, occurring in roughly eight percent of men. It is divided into acute bacterial, chronic bacterial, chronic - pelvic pain syndrome, and asymptomatic (typically found on biopsy done for a different reason); these are distinguished primarily by history and lab studies. One should also consider bladder cancer, nephrolithiasis, and enteroviscular fistula on the differential with these conditions (1).

A 14-year-old male patient, with no past medical history, presents to the clinic with 2-months of pain in his lower abdomen. Upon further questioning he states that the pain is a dull and a 2/10 in severity that worsens the longer he stands. The pain goes away when he lies down. The patient is not sexually active and has had no recent trauma. His vital signs are all within normal limits. On physical exam you note a palpable and slightly tender mass on his left testicle. An ultrasound of the mass is performed and can be seen in Figure A. What is a potential seqelae of this issue if left untreated? A) Testicular necrosis B) Testicular abscess C) Testicular atrophy D) Intestinal necrosis E) Testicular rupture

C, Varicocele This patient has a varicocele, which is a dilation of the pampiniform venous plexus in the scrotum. Prolonged, unresolved varicocele can result in testicular atrophy. Varicocele is extremely common in adolescent and adult men (~15%). It commonly occurs on the left side due to the anatomical angle affecting drainage of the left testicular vein into the left renal vein. Varicocele is associated with decreased testicular volume, abnormal semen analysis, and potentially decreased fertility. In fact, ~40% of men with infertility have a varicocele. Indications for varicocelectomy are unclear but may be recommended in patients who have pain, decreased testicular volume, or infertility. Crawford et al. discuss diagnoses of varicocele. A varicocele feels like a bag of worms upon palpation and increases in size upon straining or Valsava maneuver. It does not transilluminate with a penlight. Varicoceles are often painless but may be uncomfortable after prolonged standing. Adolescence is the most common time period of onset of varicoceles. Kolon discusses evaluation and management of varicoceles. Varicocele assessment includes grade of mass, testicular volume, endocrine evaluation, semen analysis and ultrasound. Grade (I-III) of a varicocele is as follows: I-palpable when standing and performing Valsava, II-Palpable without Valsava, III-Easily visible. The grade of the varicocele has not been associated with changes in testicular health. Decreased testicular volume, is associated with decreased spermatogenic potential and is an indication for surgery. Figure A demonstrates a dilated pampiniform plexus, indicative of varicocele. Incorrect Answers: Answer 1: Testicular necrosis can result from untreated torsion of the spermatic cord. Answer 2: Testicular abscess can result when untreated epididymitis spreads into testicular tissue. Answer 4: Intestinal necrosis can result from an inguinal hernia, which also may present as a mass in the scrotal sac. Answer 5: Testicular rupture can result from trauma.

A 6-month old boy is brought to the pediatrician's office for evaluation of a "lump" in his genital area. His birth history is unremarkable and he has been feeding and eating well. His parents report no other concerns. On physical exam, there is a tense and non-tender mass in the scrotum. The transillumination test is positive. The parents are counseled about his condition and reassured that this will most likely resolve spontaneously.

HYDROCLE(SNAPSHOT)

A 60-year-old man with a history of benign prostatic hyperplasia presents with one-sided testicular pain and urinary frequency. He also reports pain with urination. Physical exam reveals a swollen right testicle with substantial induration. Urinalysis reveals positive leukocyte esterase and 20 wbc/hpf. He is prescribed antibiotics for epididymitis.

Epididymitis (snapshot)

A 2-year-old male is brought to his pediatrician by his parents because of a lack of testes in his scrotum. Physical examination confirms that testes are absent from the scrotal sac and palpable masses are found bilaterally around the inguinal canal. If the child's condition is left untreated, levels of which of the following hormones is most likely to be decreased most when the child reaches sexual maturity? A. FSH B. LH C. Testosterone D. Inhibin E. Prolactin

Cryptorchidism is the unilateral or bilateral failure of testes to descend into the scrotal sac. This condition is associated with decreased sperm count and low levels of inhibin. Inhibin is made in the seminiferous tubules and acts as a negative regulator of FSH. In sexually mature males, the scrotum is principally important in temperature regulation during spermatogenesis. The seminiferous tubules require a cooler environment than normal body temperature to produce sperm. They can become damaged when chronically exposed to normal body temperature in cryptorchidism. Sperm count and inhibin levels are consequently decreased. This condition is managed surgically with orchidopexy, a surgery that moves undescended testicles into the scrotum and permanently fixes them there. Incorrect Answers: Answer 1: FSH levels are increased in sexually mature adults with cryptorchidism because of inhibition of negative feedback from inhibin. Answer 2-3: LH and testosterone levels are generally normal in patients with unilateral cryptorchidism, as normal body temperature does not compromise the ability of Leydig cells to produce testosterone. In bilateral cryptorchidism testosterone levels can be decreased, though it is spermatogenesis and inhibin (seminiferous tubules) are more sensitive to elevation in temperature and thus more likely to be decreased. Answer 5: Cryptorchidism does not affect prolactin production.

A 97-year-old man visits the urology clinic 5 days after experiencing urinary retention at an emergency department visit. The patient has a history of hypertension, type II diabetes mellitus, stroke, dyslipidemia, a past myocardial infarction, and severe osteoarthritis in his right hip. He is not compliant with his medications and his multiple comorbidities are poorly managed. In the hospital, the patient's urinary retention was treated with Foley catheterization. At clinic, the patient's serum-specific prostate-specific antigen (PSA) is 6.0 ng/mL (normal is < 4 ng/mL). Digital rectal examination (DRE) demonstrates a nontender prostate with several rock hard nodules. The patient's Foley is removed and he is able to urinate on his own. Which is the most appropriate next step in management? A) CT abdomen and pelvis B) Cystourethroscopy C) Transrectal prostate biopsy D) Reassurance E) Repeat PSA test

D, Prostate Cancer A transient elevation in PSA will often decrease after a period of 2-6 weeks once the underlying cause has resolved. This elderly patient should not be screened for prostate cancer given his age and multiple comorbidities despite his PSA and nodular prostate. PSA is a serine protease that liquefies semen after ejaculation. PSA has a diagnostic role in prostate cancer (though is not routinely ordered) and is also used to measure the response to cancer treatment. However, PSA is also commonly elevated in benign prostatic hyperplasia, which is common among elderly men. Transient PSA elevations can occur due to urinary retention, prostatic inflammation, as well as cystoscopy. PSA screening and digital rectal exam are no longer recommended as routine screening for prostate cancer. Similarly, the course of prostate cancer is often long and work up for elderly or critically ill patients is not indicated as they are likely to die from natural causes or other medical issues when compared to the slow course of prostate cancer. Incorrect Answers: Answer 1: CT abdomen and pelvis is used to evaluate for pelvic and retroperitoneal lymph nodes in men with prostate cancer. This patient has not yet been diagnosed with prostate cancer. Answer 2: Cystourethroscopy can be used to assess for obstruction of the bladder. The patient no longer has symptoms of urinary retention and there is not a concern for bladder cancer which presents with painless hematuria. Answer 3: Transrectal prostate biopsy would be indicated if the patient had a suspicious digital rectal exam (rock-hard nodules) and was younger with less medical comorbidities. This patient's age and other medical problems make him a poor candidate for invasive testing and procedures that may have no impact on his quality of life. Answer 5: Repeat PSA test at a later time is incorrect as it would not change management. In this elderly patient with multiple comorbidities, even if the level remains elevated, it would not change management. Bullet Summary: Prostate cancer should not be worked up in elderly patients with multiple medical comorbidities given that treatment of their prostate cancer is not likely to improve morbidity or mortality.

A 51-year-old man presents to the emergency department with an episode of syncope. He was at a local farmer's market when he fainted while picking produce. He rapidly returned to his baseline mental status and did not hit his head. The patient has a past medical history of diabetes and hypertension but is not currently taking any medications. His temperature is 97.5°F (36.4°C), blood pressure is 173/101 mmHg, pulse is 82/min, respirations are 14/min, and oxygen saturation is 98% on room air. Physical exam is notable for clear breath sounds and a S4 heart sound. Rectal exam reveals a firm and nodular prostate that is non-tender and a fecal-occult sample that is negative for blood. Which of the following is this patient's presentation most concerning for? A) Benign prostatic hyperplasia B) Normal physical exam C) Prostate abscess D) Prostate cancer E) Prostatitis

D, Prostate Cancer This patient is presenting with an incidentally discovered firm and nodular prostate which is concerning for prostate cancer. Prostate cancer screening with a digital rectal exam and a PSA is no longer routinely performed; however, it is performed occasionally in elderly patients per patient request. If a rock-hard, nodular prostate is discovered incidentally on physical exam, it is possible that the patient may have prostate cancer and should undergo further workup for their malignancy including a biopsy followed by either surgery or external radiation. If there is metastatic disease, then androgen blockade is often indicated with treatment including flutamide followed by leuprolide/goserelin. Incorrect Answers: Answer 1: Benign prostatic hyperplasia would present with a firm/smooth, enlarged prostate and may present with urinary retention. Treatment includes tamsulosin and finasteride. Answer 2: Normal physical exam would present with a normal-sized, smooth prostate that is nontender. Answer 3: Prostate abscess would present with a fever, pyuria, and a fluctuant/tender mass in the prostate. Drainage and IV antibiotics are indicated in this condition. Answer 5: Prostatitis would present with a fever, pyuria, and a soft/boggy/tender prostate. IV antibiotics are indicated in this condition. Bullet Summary: Prostate cancer presents with a firm, nodular prostate.

A 24-year-old male presents to his primary care physician with progressive chest discomfort over the last month. The discomfort is constant and sometimes associated with shortness of breath. The patient has also noticed a 12-pound unintentional weight loss over the last 6 weeks, but he believes his breasts have increased in size. He feels more fatigued than usual but denies night sweats, palpitations, and dizziness. The patient had unilateral cryptorchidism with successful orchiopexy as an infant. He also had an episode of hepatitis A during a trip to Mexico in college. He is now a graduate student, drinks socially, and occasionally smokes marijuana. His mother has type II diabetes, and his father is an alcoholic. At this visit, his temperature is 99.9°F (37.7°C), blood pressure is 121/78 mmHg, pulse is 80/min, and respirations are 14/min. There is notable bilateral gynecomastia and fullness of the anterior chest wall. His labs are shown below: Leukocyte count: 8,000/mm^3 Hemoglobin: 13.9 g/dL Platelet count: 155,000/mm^3 Serum: Aspartate aminotransferase: 12 U/L Alanine aminotransferase: 18 U/L Alkaline phosphatase: 82 IU/L Alpha-fetoprotein (AFP): 5,500 ng/mL Serum ß-hCG: 31,000 IU/L A chest radiograph is shown in Figure A. Which of the following is the best next step in management? A) Orchiectomy B) Bone marrow biopsy C) Chemotherapy D) Biopsy of mass E) Surgical removal of mass

D, Testicular Cancer This patient presents with chest discomfort, systemic symptoms, and elevated AFP and ß-hCG, most likely due to a nonseminomatous germ cell tumor (NSGCT) of the mediastinum. This combination of lab findings is considered highly suggestive for this malignancy, but biopsy is still required for definitive diagnosis before proceeding with treatment. NSGCTs include yolk sac tumor, embryonal carcinoma, and choriocarcinoma. They are more aggressive than seminomatous germ cell tumors (SGCTs) and when extragonadal, typically present with chest discomfort or dyspnea, as seen in this patient. NSGCTs frequently produce both AFP and beta-hCG, which can cause gynecomastia. As in this patient, they may appear as an anterior mediastinal mass on chest radiograph. Though there are many causes of anterior mediastinal masses ("5 T's" - thymoma, thyroid cancer/goiter, terrible lymphoma, teratoma/GCT, thoracic aorta), this patient's lab findings and gynecomastia are relatively specific for a GCT. NSGCTs may arise in the testicles and can metastasize elsewhere, but most mediastinal incidences are primary cancers rather than metastases. Testicular exam and ultrasound are still recommended to exclude testicular origin. Notably, this patient's history of cryptorchidism does place him at risk for testicular cancer, but such cases tend to be SGCT rather than NSGCT. It is unlikely, then, that this history is related to his current presentation. Hepatocellular carcinoma can also cause the AFP elevation seen in this patient, but he has no history of chronic liver disease or cirrhosis (since hepatitis A is almost exclusively acute and tends to be mild), has normal liver function tests, and is very young for liver malignancy. Extragonadal NSGCT remains the most likely diagnosis, so biopsy of the mediastinal mass should be performed. Figure A shows a mediastinal mass, likely to be a GCT. Incorrect Answers: Answer 1: Orchiectomy would be indicated if testicular cancer were confirmed and is often performed prior to chemotherapy. For this patient, his NSGCT is possibly primary as opposed to a result of metastasis from the testicles, so performing an orchiectomy without identifying a testicular source would be inappropriate. Answer 2: Bone marrow biopsy would be the next step if a hematologic malignancy were suspected. Leukemia and Hodgkin lymphoma are the most common such malignancies in a young person, and bone marrow biopsy may show abnormally proliferating white blood cells or Reed-Sternberg cells, respectively. While these can present with the systemic symptoms seen in this patient and lymphoma can cause an anterior mediastinal mass, they would not produce AFP or ß-hCG or cause gynecomastia. Furthermore, the expected hematologic abnormalities such as elevated white blood cell count or anemia and thrombocytopenia are not present in this patient. Answer 3: Chemotherapy is the first-line treatment for NSGCTs in the mediastinum and is typically completed prior to surgery. Cisplatin, etoposide, and ifosfamide are often used together, since NSGCTs are aggressive. However, without histologic confirmation of the diagnosis, proceeding with chemotherapy is not indicated. Identifying a NSGCT is important, as bleomycin is frequently used for SGCT but not NSGCT. Answer 5: Surgical removal of the mass is inappropriate without first biopsying it. In addition, neoadjuvant chemotherapy is the standard of care, with resection following for any remaining mass. Bullet Summary: In a young male with elevated ß-hCG and AFP, a large mediastinal mass is most likely to be a nonseminomatous germ cell tumor. Although testicular exam and ultrasound should be performed, the mediastinal mass is often a primary malignancy as opposed to metastasis and should be biopsied.

A 15-year-old boy presents to the emergency room with severe lower abdominal pain that awoke him from sleep about 3 hours ago. The pain is sharp and radiates to his left thigh. While in the emergency room, the patient experiences one episode of vomiting. His temperature is 99.3°F (37.4°C), blood pressure is 126/81 mmHg, pulse is 119/min, respirations are 14/min, and oxygen saturation is 99% on room air. Abdominal examination reveals no tenderness in all 4 quadrants. Scrotal examination reveals an elevated left testicle that is diffusely tender. Stroking of the patient's inner thigh on the left side does not result in elevation of the testicle. What is the next step in the management of this patient? A) CT scan of abdomen and pelvis B) IV antibiotics C) Observation and morphine D) Surgical exploration E) Testicular doppler ultrasound

D, Testicular Torsion This patient is presenting with abdominal pain, vomiting, an absent cremasteric reflex, and an elevated testicle concerning for testicular torsion. This is a surgical emergency and the patient should be taken directly to the operating room without delay in order to save the testicle. Testicular torsion presents with sudden onset and severe abdominal and testicular pain. On physical exam, the most accurate diagnostic sign is an absent ipsilateral cremasteric reflex; however, other findings include an elevated testicle with a horizontal lie. Any patient for whom there is a clinical picture supporting a diagnosis of testicular torsion should next be managed with emergency surgery. Other possible steps in the workup of testicular torsion include an ultrasound with Doppler; however, this test should never be performed if it will delay surgical intervention. Manual detorsion can be attempted if there is any delay in emergency surgery. Incorrect Answers: Answer 1: CT scan of the abdomen and pelvis could be indicated in the workup of appendicitis which presents with right lower quadrant abdominal pain with a positive Rovsing sign but would not be appropriate in this surgical emergency. Answer 2: IV antibiotics would be appropriate for epididymitis which presents with a tender posterior testicle with pain sometimes relieved by elevation of the testicle (Prehn sign). Fever and pyuria can also be seen in this condition and the pain is not sudden onset and severe. Answer 3: Observation and morphine is appropriate management if a patient is only having pain without a life-threatening diagnosis warranting treatment. Testicular torsion is a surgical emergency as the more time that passes the more likely it is that the testicle suffers permanent ischemic damage. Answer 5: Testicular Doppler ultrasound would be an appropriate confirmatory diagnostic test and could demonstrate decreased blood flow to the testicle. It would not be appropriate in a patient with a classic presentation of testicular torsion as it could delay surgical intervention. Bullet Summary: Emergency surgery is the best initial step in management in testicular torsion.

A 46-year-old woman presents to the clinic complaining that she "wets herself." She states that over the past year she has noticed increased urinary leakage. At first it occurred only during her job, which involves restocking shelves with heavy appliances. Now she reports that she has to wear pads daily because leakage of urine will occur with simply coughing or sneezing. She denies fever, chills, dysuria, hematuria, or flank pain. She has no significant medical or surgical history, and takes no medications. Her last menstrual period was 8 months ago. She has 3 healthy daughters that were born by vaginal delivery. Which of the following tests, if performed, would most likely identify the patient's diagnosis? A) Estrogen level B) Methylene blue dye C) Post-void residual volume D) Q-tip test E) Urodynamic testing

D, Urinary Incontinence The patient is presenting with urinary incontinence associated with movements that increase intra-abdominal pressure. This is consistent with stress incontinence that can be diagnosed by the Q-tip test. Stress incontinence presents with urinary leakage that follows movements that increase intra-abdominal pressure such as exercise, lifting, or coughing. Patients usually do not have more frequent urges to urinate nor urine loss at night. Stress incontinence can be secondary to a weakened pelvis floor and may be associated with multiple vaginal deliveries. On physical examination, a cystocele may be present. The Q-tip test will show increased urethral mobility. The Q-tip test is when a lubricated Q-tip is placed into the urethra and the patient is asked to cough/strain to assess for motility. Conservative first-line treatment is Kegel exercises. Refractory cases can be treated by surgery such as placement of a pessary or midurethral sling. Incorrect Answers: Answer 1: Estrogen levels will be low in menopause. While topical estrogen can be used to treat stress incontinence in postmenopausal women, measurement of estrogen levels would not be diagnostic for stress incontinence itself. Answer 2: Methylene blue dye can be instilled into the bladder to test for a vesicovaginal fistula. If a tampon placed into the vagina turns blue, this is a positive test. Patients with a vesicovaginal fistula present with painless, continuous leakage from the vagina. Answer 3: Post-void residual volume is not used for the diagnosis of overflow incontinence. Overflow incontinence presents as urine loss without warning or triggers. It is caused by incomplete bladder emptying that results in urinary leakage when the bladder is at its capacity. Answer 5: Urodynamic testing can be used to evaluate for urge incontinence. Urge incontinence is caused by detrusor muscle overstimulation. Patients present with the increased urges to urinate and inability to reach the bathroom in time and frequently have urinary leakage at night. Bullet Summary: The Q-tip test can be used to evaluate for stress incontinence.

A 75-year-old woman presents to the physician with a complaint of a frequent need to void at nighttime, which has been disrupting her sleep. She notes embarrassingly that she is often unable to reach the bathroom in time, and experiences urinary leakage throughout the night as well as during the day. The patient undergoes urodynamic testing and a urinalysis is obtained which is normal. She is instructed by the physician to perform behavioral training to improve her bladder control. Which of the following is the most likely diagnosis contributing to this patient's symptoms? A) Overflow incontinence B) Stress incontinence C) Total incontinence D) Urge incontinence E) Urinary tract infection

D, Urinary Incontinence This patient with frequent urges to void and nighttime urinary leakage that is being managed with behavioral training most likely has urge incontinence. Urge incontinence is characterized by strong, unexpected urges to void that are unrelated to position or activity and cause frequent urinary leakage that can occur at night and disrupt sleep. Urge incontinence is caused by detrusor hyperreflexia or dysfunction of the urethral sphincter muscles. Diagnosis of urge incontinence is confirmed via urodynamic testing, and treatment involves an initial management step of behavioral bladder training. If the patient's incontinence is not improved by behavioral training alone, then medical management may involve anticholinergic medications (e.g. oxybutynin) or mirabegron. Incorrect Answers: Answer 1: Overflow incontinence is characterized by chronic urinary retention and incomplete bladder emptying that leads to urinary leakage. Overflow incontinence is treated with urethral catheterization in acute settings, treatment of underlying diseases, and timed voiding. Answer 2: Stress incontinence is characterized by urinary incontinence in the setting of increased intra-abdominal pressure (e.g. coughing, sneezing, and laughing) and is treated with pelvic floor muscle strengthening, a pessary, or a mid-urethral sling in patients unresponsive to initial therapies. Answer 3: Total incontinence is characterized by uncontrolled voiding at all times and is treated with surgery. Answer 5: Urinary tract infection may cause frequent urges to void in patients but can be excluded with a urinalysis and urine culture. Bullet Summary: Urge incontinence presents with frequent urges to void that are unrelated to activity and is initially managed with behavioral bladder training.

A 31-year-old obese Caucasian female presents to the Emergency Department late in the evening for left lower quadrant pain that has progressively worsened over the last several hours. She describes the pain as sharp and shooting, coming and going. Her last bowel movement was this morning. She has also had dysuria and urgency. Her surgical history is notable for gastric bypass surgery 2 years prior and an appendectomy at age 9. She is sexually active with her boyfriend and uses condoms. Her temperature is 99.5 deg F (37.5 deg C), blood pressure is 151/83 mmHg, pulse is 86/min, respirations are 14/minute, BMI 32. On physical exam, she has left lower quadrant tenderness to palpation with pain radiating to the left groin and left flank tenderness on palpation. Her urinalysis shows 324 red blood cells/high power field. Her pregnancy test is negative. What is the next best step in management? A) A KUB (kidneys, ureters and bladder) plain film B) Intravenous pyelogram C) Transvaginal ultrasound D) Noncontrast CT scan E) Exploratory laparoscopy

D,Nephrolithiasis The clinical presentation is most likely consistent with nephrolithiasis. The most appropriate next step is to obtain a noncontrast helical CT scan to evaluate for renal stones. The patient above endorses symptoms commonly found with renal stones such as colic pain (depending on where the stone is located), dysuria, urgency, nausea and vomiting. Additionally, this patient has a history of gastric bypass surgery, which increases the risk of stone formation. The patient also shows hematuria without signs of infection on urinalysis. A noncontrast helical CT scan has high sensitivity and specificity for detecting all types of renal stones and can rule out other etiology for abdominal pain (Illustration A). Portis et al. describes that noncontrast helical CT scans have high sensitivity (95-100%) and specificity (94 to 96%) in identifying all stones types. Another advantage of this imaging modality is that measuring Hounsfield density of calculi allows one to differentiate between cysteine and uric acid stones as well as subtyping calcium containing stones. In contrast, plain films sensitivity is 45-59% and specificity is 71-77%. Plain films may not detect stones that are purely uric acid or cysteine. Asplin et al. has found that patients who have undergone modern bariatric surgery demonstrate a higher level of hyperoxaluria, which increases the risk of renal oxalate stone formation. This is likely due to the increase absorption of oxalate absorption after surgery. Illustration A: The image depicts a non-contrast CT of the abdomen and pelvis. A calculus is seen on the left ureter. Incorrect Answers: Answer 1: A plain film will detect large radiopaque stones, but may miss radiolucent stones such as uric acid or smaller size stones. Answer 2: An intravenous pyelogram has higher sensitivity and specificity for detecting stones than a plain film and can show the degree of obstruction. However, this modality has fallen out of favor due to risk of contrast reaction and radiation exposure. Answer 3: A transvaginal ultrasound may be useful if there is suspicion for ectopic pregnancies or adnexal lesions and masses. However, the patient is not pregnant, has no signs of pelvic inflammatory disease, and the clinical presentation is more consistent with renal stones. Answer 5: Exploratory laporoscopy is an invasive procedure and some type of imaging would be useful for guidance before subjecting the patient to unnecessary risk and complication.

A 16-year-old man presents to the emergency department with a 2-hour history of sudden-onset abdominal pain. He was playing football when his symptoms started. The patient's past medical history is notable only for asthma. Social history is notable for unprotected sex with 4 women in the past month. His temperature is 99.3°F (37.4°C), blood pressure is 120/88 mmHg, pulse is 117/min, respirations are 14/min, and oxygen saturation is 99% on room air. Physical exam is noted for a non-tender abdomen. Testicular exam reveals a right testicle which is elevated with a horizontal lie and the scrotum is neither swollen nor discolored. Which of the following is the most likely diagnosis? A) Appendicitis B) Epididymitis C) Seminoma D) Testicular torsion E) Traumatic urethral injury

D,Testicular torsion This patient is presenting with sudden-onset abdominal pain, a benign abdominal exam, and an elevated testicle with a horizontal lie suggesting a diagnosis of testicular torsion. Testicular torsion occurs when the testicle twists around its blood supply leading to ischemia. Acute testicular pain follows rapidly; however, other common presentations can include severe abdominal pain, nausea, and vomiting. The most common underlying cause is a congenital malformation known as a bell-clapper deformity in which the testis is inadequately affixed to the spermatic cord, allowing it to rotate freely on its axis and subsequently entangle. The diagnosis can be made clinically with a classic physical exam finding of a testicle with a horizontal lie in the setting of sudden testicular or abdominal pain. However, unclear cases can further be worked up with an ultrasound with Doppler to detect blood flow to the testicle. An ultrasound should never be performed to confirm the diagnosis if it in any way delays emergent surgical treatment. Other possible treatments can include manual detorsion which is particularly indicated when there is a delay to definitive surgical care (such as a patient in a rural hospital). Other useful findings that can clinically support a diagnosis of testicular torsion include an absent cremasteric reflex and a negative Prehn sign (improved pain with elevation of the testicle). Incorrect Answers: Answer 1: Appendicitis presents with abdominal pain that is typically periumbilical (associated with nausea/vomiting) followed by symptoms that migrate to the right lower quadrant. The diagnosis is confirmed with a CT scan and surgical removal is the typical treatment. Answer 2: Epididymitis in a young patient is typically due to sexually transmitted organisms (such as Chlamydia trachomatis and Neisseria gonorrhea in contrast to E. coli which is seen in elderly patients). These patients could present with tenderness of the posterior testicle, a fever, pyuria, and symptoms that generally are not sudden in onset. Answer 3: Seminoma is a malignant tumor that presents as a painless mass that does not transilluminate. Orchiectomy is the typical treatment. Answer 5: Traumatic urethral injury is associated with pelvic trauma and is marked by symptoms such as blood at the urethral meatus. These patients require a retrograde urethrogram prior to any attempt to pass a Foley catheter. Bullet Summary: Testicular torsion presents with sudden-onset testicular/abdominal pain and presents with an elevated testicle with a horizontal lie.

A 35-year-old woman arrives to the clinic complaining of progressive urinary leakage that has occurred for the past 1 year. At first, she would notice leakage only during athletic exercise, but now the incontinence occurs even when she laughs or coughs. The patient states that she goes to the bathroom more frequently to try to prevent "wetting myself." She wakes up once a night to urinate. She denies dysuria, hematuria, abdominal pain, and abnormal vaginal discharge. The patient has bipolar syndrome and takes lithium. She had an uncomplicated vaginal delivery 10 years ago and a cesarean section 4 years ago. She has had no other surgeries. She drinks at least 6 glasses of water a day but may drink more on days she goes for a long run. She also has a large coffee in the morning and another coffee mid-day if she "needs to focus." The patient denies tobacco, alcohol, and other recreational drug use. Pelvic examination and speculum examination are unremarkable. When that patient is asked to Valsalva, leakage of urine is observed. A urinalysis, including specific gravity, is within normal limits. A beta-human chorionic gonadotropin is negative. Which of the following is the most likely cause of the patient's symptoms? A) Diabetic polyuria B) Overactive detrusor muscle C) Primary polydipsia D) Urethral hypermobility E) Vescicovaginal fistula

D,Urinary Incontinence The patient is presenting with urinary leakage that occurs with exercise and Valsalva that is consistent with stress urinary incontinence. One cause of stress incontinence is urethral hypermobility which is characterized by the degree of change of the posterior urethrovesical angle that occurs with straining maneuvers. Stress urinary incontinence presents with urinary leakage following increased intraabdominal pressure. Common examples include coughing, physical exertion, and laughing. There is no involuntary urine loss at night. Stress incontinence can be caused by a weakened pelvic floor, increased urethral mobility, and/or poor intrinsic sphincter function. A bladder stress test involves having the patient cough or perform the Valsalva maneuver while looking for leakage of urine, and it can support the diagnosis of stress incontinence. With urethral hypermobility, instead of abdominal pressure being transmitted to the proximal urethra, abdominal pressure results in urethral movement resulting in urethral leakage. Incorrect Answers: Answer 1: Diabetic polyuria is caused by high glucose levels that result in osmotic diuresis. Glucose is excreted in the urine, and water follows glucose passively leading to high urine output. The patient's urinalysis did not detect elevated levels of glucose. Answer 2: An overactive detrusor muscle is the underlying cause of urgency urinary incontinence. Patients presents with frequent, sudden urges to urinate. Answer 3: Primary polydipsia is a psychiatric disorder that is characterized by compulsions that result in excessive fluid intake. Patients may have a concomitant psychiatric disease. Urine would show a low specific gravity. Answer 5: Vescicovaginal fistulas present with painless, continuous leakage of urine from the vagina. A fistula may form secondary to surgery, pelvic irradiation, malignancy, or prolonged labor. Bullet Summary: One common etiology of stress urinary incontinence is urethral hypermobility.

Following passage of a calcium oxalate stone, a 55-year-old male visits his physician to learn about nephrolithiasis prevention. Which of the following changes affecting urine composition within the bladder are most likely to protect against crystal precipitation? A. Increased calcium, increased citrate, increased oxalate, increased free water clearance B. Decreased calcium, increased citrate, increased oxalate, increased free water clearance C. Decreased calcium, decreased citrate, increased oxalate, increased free water clearance D. Decreased calcium, increased citrate, decreased oxalate, increased free water clearance E. Decreased calcium, increased citrate, increased oxalate, decreased free water clearance

Decreased calcium, increased citrate, decreased oxalate, and increased free water clearance are all protective against nephrolithiasis. The composition of urine is a critical factor in the process of crystal precipitation and stone formation. Increases in the concentrations of the stone-forming compounds calcium and oxalate can lead to the precipitation of these substances in the urine causing stones to form, while high fluid intake and an associated increase in free water clearance dilutes urine and is protective against stone formation. Urinary citrate binds ionized calcium, preventing stone precipitation and leading to calcium excretion. Illustration A is a CT scan of a 3mm radio-opaque calcium-containing stone in the ureter (arrow). Incorrect Answers: Answer 1: Increased urinary calcium and oxalate favor stone formation. Both compounds can precipitate in the urine when present in high concentrations. Answer 2: Increased urinary oxalate favors stone precipitation. Answer 3: Decreased urinary citrate and increased oxalate favor stone precipitation. Answer 5: Increased urinary oxalate and decreased free water clearance favor stone precipitation.

An 84-year-old man presents to the emergency department for a loss of consciousness. The patient states that he was using the bathroom when he lost consciousness and fell, hitting his head on the counter. The patient has a past medical history of diabetes, hypertension, obesity, factor V leiden, constipation, myocardial infarction, and vascular claudication. His current medications include lisinopril, atorvastatin, valproic acid, propranolol, insulin, metformin, and sodium docusate. The patient denies use of illicit substances. His temperature is 99.5°F (37.5°C), blood pressure is 167/98 mmHg, pulse is 90/min, respirations are 15/min, and oxygen saturation is 98% on room air. Physical exam reveals an elderly man sitting comfortably in his stretcher. Cardiac exam reveals a systolic murmur heard at the right upper sternal border that radiates to the carotids. Pulmonary exam reveals mild bibasilar crackles. Neurological exam reveals 5/5 strength in his upper and lower extremities with normal sensation. The patient's gait is mildly unstable. The patient is unable to give a urine sample in the emergency department and states that he almost fainted again when he tried to. Which of the following is the most likely diagnosis? A) Transient ischemic attack B) Postural hypotension C) Seizure D) Cardiac arrhythmia E) Situational syncope

E, Benign Prostatic Hyperplasia (BPH) This patient is presenting with syncope whenever he tries to urinate, suggesting a diagnosis of situational syncope secondary to benign prostatic hyperplasia (BPH). Situational syncope is a type of vasovagal syncope that only occurs during certain situations such as dehydration or during straining with bowel movements or urination in BPH. Situations that increase intra-abdominal pressure can lead to a vagal response that causes sudden bradycardia and hypoperfusion of the CNS resulting in syncope. Patients presenting with syncope should be worked up for a cardiac or neurologic process before making the diagnosis of situational syncope. Incorrect Answers: Answer 1: Transient ischemic attack presents with sudden stroke-like symptoms (loss of vision and unilateral weakness) that typically self-resolve by the time the patient presents. This patient's symptoms are reproducible with urination, suggesting an alternate diagnosis. Answer 2: Postural hypotension presents with syncope when a patient stands up suddenly. A slow increase in vascular tone results in temporary blood pooling in the lower extremities and hypoperfusion of the CNS which can cause syncope. This patient's symptoms only occur when he tries to urinate. Answer 3: Seizure presents with symptoms during the episode and a post-ictal state. This patient does not seem to have a post-ictal state and has symptoms reproducible with urination. Answer 4: Cardiac arrhythmia is a possible diagnosis in this patient, and an EKG should be performed to rule out a cardiac diagnosis. In the setting of syncope specific to certain situations, situational syncope is a more likely diagnosis. Bullet Summary: Situational syncope is a type of vasovagal syncope that only occurs in certain circumstances (most commonly dehydration or straining).

A 23-year-old woman presents to the emergency department with abdominal pain. The patient states that she has pain in her back that radiates to her abdomen. The pain started yesterday and is episodic. She describes the pain as a 10/10 when it occurs. The patient has a past medical history of asthma, anxiety, and chlamydia that was appropriately treated. Her current medications include fluoxetine. The patient's lab values are ordered as seen below. Hemoglobin: 14 g/dL Hematocrit: 40% Leukocyte count: 8,500 cells/mm^3 with normal differential Platelet count: 225,000/mm^3 Serum: Na+: 140 mEq/L Cl-: 100 mEq/L K+: 4.9 mEq/L HCO3-: 24 mEq/L BUN: 22 mg/dL Glucose: 89 mg/dL Creatinine: 1.2 mg/dL Ca2+: 10.0 mg/dL AST: 11 U/L ALT: 11 U/L Urine: Color: amber Nitrites: negative Na+: 12 mmol/24 hours Beta-hCG: positive Red blood cells: 11/hpf Which of the following is the best test for this patient's most likely diagnosis? A) Abdominal radiograph B) Abdominal CT C) Transvaginal ultrasound D) Abdominal ultrasound E) Renal ultrasound

E, Nephrolithiasis This patient is presenting with colicky abdominal pain that radiates to the abdomen in the setting of hematuria, suggesting a diagnosis of nephrolithiasis. The best test in the setting of this patient's positive urinary beta-hCG is a renal ultrasound. Nephrolithiasis presents with colicky flank pain that can radiate to the abdomen or groin. Symptoms are reproducible when the physician percusses the flanks. Patients will also present with gross or microscopic hematuria. The most accurate test for nephrolithiasis is a CT scan of the abdomen. In pregnant patients, the best diagnostic test is a renal ultrasound to avoid exposing the fetus to radiation. Once the diagnosis is confirmed, management includes supportive measures including hydration and analgesia as the patient waits for the stone to pass. Incorrect Answers: Answer 1: Abdominal radiographs would not reveal enough detail to confirm the diagnosis of nephrolithiasis nor would they be indicated in a patient with a positive urine beta-hCG. Answer 2: Abdominal CT is the most accurate test for nephrolithiasis; however, it would be relatively contraindicated in a patient with a positive urine beta-hCG. Answer 3: Transvaginal ultrasound would be the diagnostic test of choice if an ectopic pregnancy was suspected. Answer 4: Abdominal ultrasound would be the best test for a diagnosis of cholelithiasis. Bullet Summary: Nephrolithiasis presents with colicky flank pain and hematuria, and the best diagnostic test in a pregnant patient is ultrasound.

A 37-year-old woman presents with an inability to void in the hours after giving birth to her first child via vaginal delivery. Her delivery involved the use of epidural anesthesia as well as pelvic trauma from the use of forceps. She is currently experiencing urinary leakage and complains of increased lower abdominal pressure. Which of the following is the most appropriate treatment for this patient's condition? A) Antimuscarinic drugs B) Midurethral sling C) Pelvic floor muscle strengthening D) Pessary insertion E) Urethral catheterization

E, Urinary Incontinence This patient who recently underwent vaginal delivery with epidural anesthesia and is now experiencing urinary leakage has overflow incontinence, which can be treated with urethral catheterization. Overflow incontinence is a condition of chronic urinary retention, featuring a chronically distended bladder due to detrusor underactivity or outlet obstruction that leads to urinary leakage with overfilling. Placement of a urethral catheter may be necessary in acute settings to allow the patient to void. The diagnosis of overflow incontinence can be made with postvoid residual measurements and urodynamic testing. Overflow incontinence is associated with neurogenic bladder (e.g. multiple sclerosis), bladder outlet obstruction (e.g. benign prostatic hyperplasia in males), and polyuria (e.g. diabetes patients). Incorrect Answers: Answer 1: Antimuscarinic drugs, such as oxybutynin, are used in the treatment of urge incontinence which is caused by detrusor muscle overstimulation and presents with frequent urges to void and urinary leakage that disrupts sleep. Answer 2: A midurethral sling can be utilized in patients with stress incontinence who are unresponsive to first-line therapy (strengthening pelvic floor muscles) and pessary. Answer 3: Pelvic floor muscle strengthening is the first-line treatment option for patients experiencing stress incontinence, which presents with urinary incontinence in response to increased intra-abdominal pressure (e.g. coughing, sneezing, and laughing). Answer 4: Pessary insertion is a treatment option for patients with stress incontinence. Bullet Summary: Overflow incontinence, which can occur after vaginal delivery, may be treated with catheterization.

A 20-year-old man presents with left-sided testicular pain and swelling. He has a past medical history of multiple sexually transmitted infections. On physical exam, his left scrotum and testicle are erythematous, indurated, and tender to palpation. An ultrasound reveals a thickened epididymis. He is sent to the lab for testing for sexually transmitted diseases.

EPIDIDYMITIS(SNAPSHOT)

A newborn male is found to have the urethral opening on the ventral side of his penis. Figure A demonstrates this defect in an adult. This abnormality is a result of which of the following? A. Incomplete fusion of the urachal duct B. Incomplete fusion of the urethral folds C. Incomplete fusion of the labioscrotal swellings D. Dysgenesis of the genital tubercle E. Vitelline fistula

Hypospadias is a congenital anomaly resulting from incomplete fusion of the urethral (urogenital) folds. Male and female genitalia arise from common precursor embryonic structures. In males, the urethral folds fuse to form the ventral wall of the penis and the penile raphe. In females, the urethral folds do not fuse, but remain separated forming the labia minora. Similarly, in the male with hypospadias, the urethral folds remain inappropriately separated, leading to the urethral opening on the ventral aspect of the penis. An important high yield association for Step 1/2 is to not circumcise patients with hypospadias as the tissue removed is often used in the reconstruction of the defect. O'Brien et al. discuss the treatment of hypospadias. They recommend surgical reconstruction early, when the child is between six and nine months old. If cryptorchidism is present in addition to the hypospadias, further investigation is necessary in order to exclude the possibility of an intersex disorder. van der Zanden et al., in a study of 712 patients with hypospadias, found that hypospadias are caused by interactions between specific genes, estrogen exposure, and maternal hypertension or preeclampsia. Illustration A shows the fusion of the urethral folds in the normal embryological development of the penis. Illustration B depicts three potential locations of hypospadias. Image A shows hypospadias in an adult. In this image it is clear that the urethral opening is on the ventral side of the penis. Incorrect answers: Answer 1: The urachal duct is a duct extending from the yolk sac to the apex of the bladder during embryonic development. If the urachal duct fails to close, a patent urachus results, and urine leaks from the umbilicus. Answer 3: The labioscrotal swellings form the scrotum in the male and the labia majora in the female. Answer 4: The genital tubercle becomes the penis in the male and the clitoris in the female. Incorrect positioning of the genital tubercle during 5th week of gestation results in epispadias, a urethral opening on the dorsal surface of the penis. Answer 5: The vitelline duct connects the yolk sac to the midgut and closes during the 7th week of embryonic development. Failure of the duct to close results in a vitelline fistula, characterized by meconium discharge from the umbilicus.

A 60-year-old man presents to his primary care physician for evaluation of erectile dysfunction. His wife died 5 years ago and he would like to start dating again. He denies having any life stressors. He reports having no morning or night-time erections. His past medical history includes hyperlipidemia managed with medication and pre-diabetes managed with an active lifestyle and diet. His physician sends him for further laboratory workup and counsels him on the use of sildenafil.

Impotence / Erectile Dysfunction (snapshot)

A 35-year-old man with a past medical history of depression presents to his primary care physician for evaluation of difficulty maintaining an erection. He is married, has 3 children all under the age of 5, and was recently fired from a long-standing job. He reports feeling constantly fatigued and losing interest in activities he previously enjoyed, such as golfing. He reports never feeling like engaging in sexual intercourse, but feels pressure to do so from his wife. He reports having daily morning erections. His routine labs from 6 months ago at an annual visit reveal normal hemoglobin A1c and lipid levels.

Impotence / Erectile Dysfunction(SNAPSHOT)

A 30-year-old man presents to the emergency department with severe, colicky left flank pain of acute onset that radiates to the left groin. He also reports urinary frequency and urgency. Physical examination is significant for left-sided costovertebral angle tenderness. Urine dipstick is heme positive. Urinalysis also shows envelope-shaped crystals. A non-contrast abdominal CT is shown.

Nephrolithiasis (snapshot)

A 36-year-old woman presents to the emergency department with severe and colicky left flank pain that radiates to the left groin. She reports increased urinary frequency and urgency. She is sexually active with multiple men and uses condoms inconsistently. On physical exam, there is left-sided costovertebral angle tenderness and lower abdominal tenderness.

Nephrolithiasis(snapshot)

A 65-year-old man presents to his physician's office after noticing blood in his urine earlier this morning. This has never happened before and he denies any new medications or prior infection. He also reports to having flank pain for the past few weeks. Medical history is significant for hypertension. He has a 40 pack-year smoking history. On physical examination there is a firm, nontender, and homogeneous mass in the right flank. Computerized tomography (CT) scan of the abdomen is shown.

Renal Cell Carcinoma (snapshot)

A 65-year-old man with a 40 pack-year history of smoking presents with right costovertebral angle pain, hematuria, fever, and a palpable mass in the right lower quadrant. CT of the abdomen demonstrates a contrast enhancing lesion on the kidney.

Renal Cell Carcinoma(Snapshot)

A 31-year-old male presents to his primary care physician complaining of low back pain and fevers. He reports a four-day history of intermittent fevers, chills, and low back pain. He denies trauma or recent illness. His past medical history is notable for recurrent renal stones, diabetes mellitus, and alcohol abuse. He takes metformin but admits to missing several doses. He has had multiple sexual partners and uses condoms intermittently. His temperature is 100.6°F (38.1°C), blood pressure is 110/70 mmHg, pulse is 110/min, and respirations are 21/min. On examination, he demonstrates mild tenderness to palpation along his lower back. Sensation to touch is intact in the bilateral lower extremities. Strength in leg and hip flexion and extension is 5/5 bilaterally. The physician is unable to perform a digital rectal examination due to pain. A urinalysis demonstrates leukocytes. Which of the following treatment regimens is most appropriate in this patient? A. Piperacillin and tazobactam B. Ampicillin and gentamicin C. Ciprofloxacin and trimethroprim-sulfamethoxazole D. Terazosin E. Ceftriaxone and doxycycline

The patient in this vignette presents with fever, chills, low back pain, and prostatic tenderness suggestive of acute prostatitis. Acute prostatitis is most commonly caused by N. gonorrhea and C. trachomatis, so the first-line treatment is ceftriaxone and doxycycline. Acute prostitis often affects young sexually active adults and presents with fever, chills, low back pain, dysuria, and urinary frequency. Physical examination will generally demonstrate an exquisitely tender, fluctuant, and/or swollen prostate on digital rectal examination. Serologic analysis may reveal leukocytosis and urinalysis may reveal pyuria, hematuria, and bacteriuria. Since it is most frequently caused by N. gonorrhea and C. trachomatis in young adults, first-line therapy involves antibiotics targeted against these organisms: ceftriaxone and doxycycline. Incorrect Answers: Answer 1: Piperacillin/tazobactam is a combination antibiotic that contains an extended-spectrum penicillin (piperacillin) and beta-lactamase inhibitor (tazobactam). It has broad coverage for many gram-positive and gram-negative infections. It is one of the first-line therapies for acute pyelonephritis but is not used to treat acute prostatitis. Answer 2: Ampicillin and gentamicin are commonly used together to provide broad-spectrum coverage for serious gram-positive and gram-negative infections. This combination is one of the first-line treatments for acute pyelonephritis. However, it is not used to treat acute prostatitis. Answer 3: Ciprofloxacin and trimethoprim-sulfamethoxazole are fluoroquinolone and sulfa-based antibiotics, respectively. This combination is used to treat acute prostatitis in older individuals (>35 years old). Answer 4: Terazosin is an alpha-1-adrenergic antagonist. It is used to provide symptomatic relief to patients with chronic prostatitis but it is not used in acute prostatitis. Alpha-blockers improve symptoms by reducing urinary obstruction due to prostate enlargement. Bullet Summary: Treatment of acute prostatis in patients < 35 years of age involves ceftriaxone and doxycycline to target N. gonorrhea and C. trachomatis.

A 24 year-old male presented to his physician with a homogenous, painless testicular swelling that he noticed for the past two weeks. After ultrasound and blood tests, he subsequently underwent a radical inguinal orchiectomy and pathology was consistent with seminoma. Which of the following would you expect to see in a tissue sample from the patient's orchiectomy? A. Lobules of large cells with prominent nuclei and watery cytoplasm B. Schiller-Duval bodies C. Reinke crystals D. Tubular papillary architecture with necrosis E. Mature derivatives of endoderm, mesoderm, and ectoderm

Seminoma tumor histology consists of lobules of large cells with a 'fried egg' appearance (large nuclei with prominent nucleoli and watery cytoplasm). Seminomas are the most common testicular tumor in males. Though malignant, seminomas have an excellent prognosis if treated early. Seminomas spread first to the para-aortic lymph nodes and then hematogenously. The tumor is treated with orchiectomy followed by radiation if there is suspicion that the disease has spread outside of the testes. Shaw reviews the diagnosis and treatment of testicular cancer. Testicular cancer is the most common cancer found in men ages 20-35. It typically presents as a painless mass or swelling in the scrotum. The primary treatment for testicular cancers is radical inguinal orchiectomy with removal of the testicle and spermatic cord. Subsequent management depends on cancer staging and specific tumor type. Bosl et al. review testicular germ cell cancers. Germ cell tumors are classified as either seminomatous or nonseminomatous. Following serum tumor marker concentrations can be useful for assessing response to treatment and monitoring for recurrence. Alpha-fetoprotein is associated with nonseminomatous germ-cell tumors. Whereas increases in hCG can be seen in both seminomatous and nonseminomatous tumors. Illustration A demonstrates seminoma histology. Incorrect answers: Answer 2: Schiller-Duval bodies resemble glomeruli and are found in Yolk sac tumors, the most common testicular tumor in children. Answer 3: Reinke crystals are rod-shaped inclusions often made of lipofuscin. They are found in Leydig cell tumors. Answer 4: In the context of testicular swelling, papillary architecture with necrosis is indicative of embryonal carcinoma. Embryonal carcinomas present most frequently in mixed germ cell tumors. Answer 5: Mature teratomas frequently include derivatives from all three germ layers. Though most often benign in females, in males these tumors are generally malignant.

A 14-year-old boy is brought to the emergency room for an acute onset of testicular pain. The sharp pain started 2 hours ago on the right side of his scrotum. He also reports nausea and vomiting associated with the pain. Physical exam reveals the right scrotum to be erythematous and swollen. The cremasteric reflex is absent. He is immediately rushed into surgery for detorsion.

TESTICULAR TORSION(SNAPSHOT)

A 33-year-old man presents to his physician's office after noticing a mass on his left testicle while showering. He reports this has never happened before and denies any pain or discomfort. On physical examination, there is a firm and fixed mass on the left testicle. Bilateral scrotal ultrasound is performed and significant findings are shown.

Testicular Cancer (snapshot)

A neonatal boy is brought to his pediatrician for concern of a blue mass in the scrotum. His parents noticed this an hour ago. On physical exam, the scrotum is blue and firm with some erythema. Transillumination test is negative. Doppler ultrasound shows absent blood flow. The neonate is immediately sent to hospital for surgery.

Testicular Torsion (snapshot)

A 38-year-old woman at 24 weeks' gestation undergoes prenatal ultrasonography. Results show a male fetus with bilateral hydrouretero-nephrosis. Soon after birth, laboratory values show a moderately elevated creatinine. A voiding cystourethrogram reveals posterior urethral valves in the prostatic urethra. The etiology of posterior urethral valves remains unclear, but failure of regression of the developmental structure that comprises the prostatic urethra has been postulated. Anomalous development of what structure may give rise to posterior urethral valves? (A) Genital tubercle (B) Mesonephric duct (C) Paramesonephric duct (D) Urogenital folds (E) Urogenital sinus

The correct answer is E. The urogenital sinus in the male gives rise to the bladder, prostate, prostatic and membranous parts of the urethra and bulbourethral glands. Answer A is incorrect. The genital tubercle gives rise to the glans penis and corpus spongiosum in the male, or the glans clitoris in the female. In contrast, posterior urethral valves may be due to failure of regression of the urogenital sinus, which normally gives rise to the prostatic urethra. Answer B is incorrect. The mesonephric (wolffian) duct develops into the seminal vesicles, epididymis, ejaculatory duct, and vas deferens. In contrast, posterior urethral valves may be due to failure of regression of the urogenital sinus, which normally gives rise to the prostatic urethra. Answer C is incorrect. The paramesonephric (müllerian) duct develops into the fallopian tube, uterus, and superior third of the vagina in the female. In contrast, posterior urethral valves may be due to failure of regression of the urogenital sinus, which normally gives rise to the prostatic urethra. Answer D is incorrect. The urogenital folds make up the ventral shaft of the penis and penile urethra in the male. In contrast, posterior urethral valves may be due to failure of

A 20-year-old healthy male presents to his primary care physician complaining of painless testicular swelling. On physical examination, his left testicle is enlarged and nontender. An ultrasound of his left testicle reveals a homogenous solid mass. He subsequently undergoes a radical inguinal orchiectomy. A histologic slide of the specimen is shown in Figure A. Which of the following serum markers was most likely elevated in this patient? A. Cancer antigen 125 B. Cancer antigen 19-9 C. Placental alkaline phosphatase D. Alpha-fetoprotein E. ß-human chorionic gonadotropin

The most likely diagnosis in this patient is a testicular seminoma. Seminomas have a "fried egg" appearance on histology and are often associated with an elevation in serum placental alkaline phosphatase (PLAP). In this young healthy male patient with a unilateral testicular mass, the differential diagnosis includes malignant tumors of germ cell and non-germ cell origins. Seminoma is the most common testicular tumor and is characterized by classic "fried egg" appearance on histology. Seminoma is a germ cell tumor that frequently affects males aged 15-35 years and is analogous to dysgerminoma of the ovary. Clinically, seminomas present with painless unilateral homogenous testicular enlargement. Seminomas have a good prognosis because they metastasize late and are very radiosensitive. PLAP is often elevated in seminoma patients. Figure A demonstrates the characteristic "fried egg" appearance of seminomas. Note the large cells in lobules with watery cytoplasm. Incorrect Answers: Answer 1: Cancer antigen 125 (CA-125) is normally released by inflamed peritoneal tissue. It is associated with epithelial ovarian tumors and is used to monitor response to therapy and detect recurrence in patients with ovarian tumors. Answer 2: Cancer antigen 19-9 (CA 19-9) is associated with pancreatic adenocarcinoma. It is used to monitor response to therapy in patients with known pancreatic adenocarcinoma. Answer 4: Alpha-fetoprotein (AFP) is elevated in yolk sac (endoderminal sinus) tumors. These are germ cell testicular tumors that commonly affect children under 4 years. Answer 5: ß-human chorionic gonadotropin (ß-hCG) is normally released by the corpus luteum in early pregnancy. Choriocarcinomas are a type of testicular germ cell tumor that secrete ß-hCG. Bullet Summary: Seminomas are the most common testicular tumor and are characterized by a "fried egg" appearance on histology. Serum placental alkaline phosphatase is often elevated in seminoma patients.

A 55-year-old male presents to his primary care physician complaining of dark urine. He first noticed that his urine was darker than usual approximately two months ago. He denies dysuria, urgency, or hesitancy. His past medical history is notable for a history of severe rheumatoid arthritis requiring multiple long-term anti-inflammatory medications. He has otherwise been healthy and takes no other medications. He has never smoked and does not drink alcohol. His temperature is 98.6°F (37°C), blood pressure is 130/85 mmHg, pulse is 80/min, and respirations are 18/min. He has no costovertebral angle tenderness. Urinalysis is positive for heme. A CT urogram is shown in Figure A. Which of the following medications has this patient likely taken in the past? A. Cyclophosphamide B. Cyclosporine C. Hydroxychloroquine D. Azathioprine E. Etanercept

The most likely diagnosis in this patient is transitional cell carcinoma (TCC) of the bladder. Risk factors for TCC include cyclophosphamide, aniline dyes, nitrosamines, phenacetin, and smoking. In this patient with painless hematuria, a malignancy of the genitourinary tract should be suspected. The CT urogram in Figure A demonstrates a filling defect in the bladder, suggesting that this patient has bladder cancer. TCC is the most common type of bladder cancer and frequently arises on the lateral or posterior walls of the bladder. It can also arise in the renal calyces, renal pelvis, and ureters. Multiple substances and medications increase the risk of TCC. These include cyclophosphamide, aniline dyes, nitrosamines, phenacetin, and cigarette smoke. Given that this patient has a history of severe rheumatoid arthritis, it is likely that he took cyclophosphamide as an anti-inflammatory agent. Cyclophosphamide is a DNA alkylating agent that is used to treat severe rheumatologic disorders, testicular cancer, ovarian cancer, breast cancer, and lymphomas. The risk of cyclophosphamide-associated TCC is dose-dependent and therefore increases with chronic use. Figure A is an image from a CT urogram. A filling defect can be seen on the left lateral wall of the bladder, indicating the presence of a mass within the bladder. Incorrect Answers: Answer 2: Cyclosporine is an immunosuppressive agent that is associated with the development of lymphoproliferative disorders. Answer 3: Hydroxychloroquine is a disease-modifying anti-rheumatic drug (DMARD) that is used to treat rheumatoid arthritis. It is also used as an antimalarial medication. Hydroxychloroquine is not associated with TCC. Answer 4: Azathioprine is an immunosuppressive agent that is associated with an increased risk of developing several cancers including non-Hodgkin lymphoma, squamous cell carcinoma of the skin, and hepatobiliary carcinomas. Answer 5: Etanercept is a TNF-alpha decoy receptor that is used to treat autoimmune diseases including ankylosing spondylitis, psoriasis, psoriatic arthritis, and rheumatoid arthritis. It is not associated with TCC. Bullet Summary: Transitional cell carcinoma is the most common form of bladder cancer. Cyclophosphamide, aniline dyes, nitrosamines, phenacetin, and smoking all increase the risk of developing transitional cell carcinoma.

A 74-year-old male presents to his primary care physician complaining of left lower back pain. He reports a four-month history of worsening left flank pain. More recently, he has started to notice that his urine appears brown. His past medical history is notable for gout, hypertension, hyperlipidemia, and myocardial infarction status-post stent placement. He has a 45 pack-year smoking history and drinks 2-3 alcoholic beverages per day. His temperature is 100.9°F (38.3°C), blood pressure is 145/80 mmHg, pulse is 105/min, and respirations are 20/min. Physical examination is notable for left costovertebral angle tenderness. A CT of this patient's abdomen is shown in figure A. This lesion most likely arose from which of the following cells? A. Proximal tubule cells B. Distal convoluted tubule cells C. Mesangial cells D. Perirenal adipocytes E. Collecting duct epithelial cells

The patient in this vignette presents with costovertebral angle pain, hematuria, a significant smoking history, and a renal mass suggestive of renal cell carcinoma (RCC). RCC is a neoplasm originating from the proximal tubule. The presence of a left renal mass on imaging in a patient with a significant smoking history makes RCC the most likely diagnosis. RCC most commonly results from malignant transformation of cells of the proximal tubule. Histologically, RCC looks like polygonal clear cells with abundant cytoplasm. Clinically, RCC presents with hematuria, flank pain, a palpable abdominal mass, and fever. It is also associated with paraneoplastic polycythemia (due to EPO production) and hypercalcemia (due to PTH-related peptide production). Figure A demonstrates a heterogeneously enhancing mass arising from the left kidney with areas of necrosis. Incorrect Answers: Answer 2: The distal convoluted tubule actively reabsorbs sodium and chloride, therefore diluting urine before it enters the collecting duct. PTH also acts on this segment to promote calcium reabsorption. RCC does not typically originate from the distal convoluted tubule. Answer 3: The renal mesangium is the supportive structure inside the glomerulus between the capillaries. Renal mesangial cells regulate capillary flow through the glomerulus. RCC does not typically originate from the renal mesangium. Answer 4: The adipose capsule of the kidney (perinephric fat) lies between the renal fascia and renal capsule. This is relevant clinically because it protects the kidney from blunt trauma. The perinephric fat also contains brown adipose tissue that play an important role in thermoregulation, especially in newborns. Answer 5: The renal collecting duct reabsorbs water to concentrate urine from the distal convoluted tubule. It is regulated by antidiuretic hormone (ADH), which increases water reabsorption by promoting aquaporin insertion. It is also regulated by aldosterone. Bullet Summary: Renal cell carcinoma is a malignant neoplasm arising from cells of the proximal tubule. It presents with hematuria, flank pain, a palpable abdominal mass, and multiple paraneoplastic syndromes (e.g., polycythemia and hypercalcemia).

A 63-year-old woman presents to her primary care doctor with increased urinary frequency. She has noticed that over the past 6 months, she has had to urinate more often than usual. Several times per day, she develops a rapid-onset need to urinate and has occasionally been unable to reach the restroom. These symptoms have caused her a lot of distress and impacted her work as a grocery store clerk. She has tried pelvic floor exercises, decreasing her caffeine consumption, and has intentionally lost 20 pounds in an effort to alleviate her symptoms. She denies urinary hesitancy or hematuria. Her past medical history is notable for rheumatoid arthritis for which she takes methotrexate. She does not smoke or drink alcohol. Her temperature is 98.8°F (37.1°C), blood pressure is 124/68 mmHg, pulse is 89/min, and respirations are 19/min. She is well-appearing and in no acute distress. Which of the following interventions would be most appropriate in this patient? A. Intermittent catheterization B. Oxybutynin C. Pessary placement D. Tamsulosin E. Topical estrogen

The patient in this vignette presents with frequent urinary leakage and an inability to reach the bathroom in time suggestive of urge incontinence. Antimuscarinic medications such as oxybutynin can be used to treat urge incontinence. Urge incontinence is one of four primary types of incontinence - others include stress incontinence, overflow incontinence, and mixed incontinence. Urge incontinence stems from detrusor muscle overstimulation and will commonly present in older women with frequent, rapid-onset urges to urinate. These episodes often occur at night, may awake the patient from sleep, and lead to an inability to reach the bathroom in time to urinate. Antimuscuarinic medications such as oxybutynin and tolterodine decrease the strength and frequency of detrusor muscle spasms and may alleviate symptoms in patients with urge incontinence. Incorrect Answers: Answer 1: Intermittent catheterization can be used to treat overflow incontinence. Overflow incontinence results from incomplete bladder emptying, so clean intermittent catheterization can drain residual urine and decrease urinary leakage. Answer 3: Pessary placement can be used to treat stress incontinence. Stress incontinence results from weakened pelvic floor muscles, poor intrinsic sphincter function, and increased urethral mobility. A pessary is a a medical device inserted into the vagina to support the pelvic organs. Answer 4: Tamsulosin is a selective alpha-1 adrenergic antagonist that can be used to treat hypertension and urinary retention in men with benign prostatic hyperplasia. Answer 5: Topical estrogen can be used to relieve some symptoms associated with incontinence in post-menopausal women. Bullet Summary: Anti-muscarinic drugs such as oxybutynin can be used to treat urge incontinence by decreasing detrusor muscle contractions.

A 76-year-old African American man presents to his primary care provider complaining of urinary frequency. He wakes up 3-4 times per night to urinate while he previously only had to wake up once per night. He also complains of post-void dribbling and difficulty initiating a stream of urine. He denies any difficulty maintaining an erection. His past medical history is notable for non-alcoholic fatty liver disease, hypertension, hyperlipidemia, and gout. He takes aspirin, atorvastatin, enalapril, and allopurinol. His family history is notable for prostate cancer in his father and lung cancer in his mother. He has a 15-pack-year smoking history and drinks alcohol socially. On digital rectal exam, his prostate is enlarged, smooth, and non-tender. Which of the following medications is indicated in this patient? A. Clonidine B. Hydrochlorothiazide C. Midodrine D. Oxybutynin E. Tamsulosin

The patient in this vignette presents with increased urinary frequency, hesitancy, and a firm enlarged prostate on exam suggestive of benign prostatic hyperplasia (BPH). The first-line treatment for BPH is alpha-1-adrenergic blockers such as tamsulosin. BPH is a common cause of lower urinary tract symptoms including urgency, frequency, hesitancy, post-void dribbling, and nocturia. The diagnosis of BPH is based on clinical history and physical exam. The first-line treatment for most patients with BPH involves alpha-1-adrenergic inhibitors such as tamsulosin and terazosin. Other medical options for treating BPH include 5-alpha-reductase inhibitors (e.g., finasteride) and phosphodiesterase-5 inhibitors (e.g., sildenafil), though alpha-1-adrenergic inhibitors should be trialed first in most patients. Incorrect Answers: Answer 1: Clonidine is an alpha-2 agonist that reduces brainstem vasomotor center-mediated central nervous system activation. It can be used as an antihypertensive and a sedative. Answer 2: Hydrochlorothiazide is a thiazide diuretic that functions by inhibiting sodium and chloride reabsorption from the distal tubule by blocking the Na-Cl symporter. It can be used as an antihypertensive and as a diuretic to address peripheral edema. Answer 3: Midodrine is an alpha-1 agonist that increases peripheral vascular tone. It can be used as a vasopressor but is not indicated in the treatment of BPH. Answer 4: Oxybutynin is an anticholinergic medication that antagonizes the muscarinic acetylcholine receptors. It can be used to treat urinary frequency, urge incontinence, and hyperhidrosis. Bullet Summary: Alpha-1-adrenergic inhibitors such as tamsulosin and terazosin are the first-line treatment for benign prostatic hyperplasia.

A 71-year-old African American man with a history of prostatic adenocarcinoma presents to his oncologist with low back pain. He was diagnosed with non-resectable prostatic adenocarcinoma 4 years ago. He has undergone radiation therapy and chemotherapy. Over the past 3 months, he has developed constant mild non-radiating low back pain that occasionally wakes him up from sleep. He denies any recent falls or trauma. His past medical history is notable for hypertension, diabetes mellitus, coronary artery disease, and gout. He also has a history of thyroid cancer and underwent thyroidectomy 5 years ago. He takes lisinopril, metoprolol, aspirin, metformin, and allopurinol. He has a 40-pack-year smoking history and drinks alcohol socially. His temperature is 99.2°F (37.3°C), blood pressure is 150/85 mmHg, pulse is 84/min, and respirations are 18/min. On exam, he is well-appearing and in no acute distress. He is mildly tender to palpation along the lumbar vertebral spinous processes. A computerized tomography (CT) scan of the lumbar spine demonstrates a blastic and sclerotic lesion in the L5 vertebral body. Which of the following findings would most likely be seen on a serum study of this patient? A. Decreased calcium, decreased phosphate, increased alkaline phosphatase, and increased parathyroid hormone B. Decreased calcium, increased phosphate, increased alkaline phosphatase, and increased parathyroid hormone C. Increased calcium, decreased phosphate, increased alkaline phosphatase, and increased parathyroid hormone D. Normal calcium, normal phosphate, increased alkaline phosphatase, and normal parathyroid hormone E. Normal calcium, normal phosphate, normal alkaline phosphatase, and normal parathyroid hormone

The patient in this vignette with a history of prostatic adenocarcinoma presents with low back pain and imaging findings consistent with lumbar spine metastases. Bone metastases from prostate cancer may present with isolated elevations in alkaline phosphatase. Prostatic adenocarcinoma is the most common cancer in men and is associated with increasing age, family history, smoking, and African American race. Prostate cancer often spreads hematogenously via the Batson venous plexus and most frequently metastasizes to bone. The most common sites of metastasis are the spine, femur, and pelvis. Bone metastases from prostatic adenocarcinoma may demonstrate isolated elevations in serum alkaline phosphatase and are frequently sclerotic, in contrast to several other cancer types which may present with lytic lesions. Incorrect Answers: Answer 1: Decreased calcium, decreased phosphate, increased alkaline phosphatase, and increased parathyroid hormone is consistent with serological findings seen in rickets or osteomalacia. Answer 2: Decreased calcium, increased phosphate, increased alkaline phosphatase, and increased parathyroid hormone is consistent with serological findings seen in secondary hyperparathyroidism. Answer 3: Increased calcium, decreased phosphate, increased alkaline phosphatase, and increased parathyroid hormone is consistent with serological findings seen in primary hyperparathyroidism. Answer 5: Normal calcium, normal phosphate, normal alkaline phosphatase, and normal parathyroid hormone (normal serum findings) are seen in some metabolic bone conditions including osteoporosis and osteopetrosis; however, blastic metastatic lesions due to prostate cancer will likely present with elevated alkaline phosphatase. Bullet Summary: Prostate cancer most frequently metastasizes to bone and may present with elevated serum alkaline phosphatase.

71-year-old male presents to your office with painless hematuria and left costovertebral angle pain. His body mass index is 33 kg/m^2. A renal biopsy is shown in Figure A. The patient's mass is most likely a malignancy of which of the following: A. Tubular epithelial cells B. Perirenal adipose cells C. Metastatic adrenocortical carcinoma D. Mesangial cells E. Visceral epithelial cells

The patient presents with classic symptoms of renal cell carcinoma (RCC), including painless hematuria and costovertebral angle pain. Renal cell carcinoma is a malignancy of proximal tubular epithelial cells. RCC symptoms include painless hematuria, costovertebral angle pain, fever and weight loss. Smoking and obesity are risk factors. The classic triad of Histology shows rounded or polygonal clear cells with abundant clear cytoplasm filled with glycogen and lipid (see image in questions stem). Prognosis is poor, especially if there is involvement of the renal vein. When approaching this question making the correct diagnosis hinges almost entirely on knowing the histology of RCC. Given the history of painless hematuria a malignancy must be the first thing that comes to mind, particularly without a history of trauma, or a congenital defect. Despite this, the classic triad of RCC (flank pain, hematuria, and a palpable abdominal renal mass) occurs in at most 9 percent of patients. Classically the cells seen in RCC and the description given on Step exams is that of rounded, polygonal cells, abundant in lipids and glycogen. The clear appearance of the cells is a result of lipids and glycogen washing out during preparation for histology. Once the diagnosis of RCC is made, the next step is knowing that this malignancy is of proximal tubular epithelial cells. Keep in mind that a major risk factor for RCC includes smoking - carcinogens are concentrated in the urinary tract and bathe the tubular epithelial cells, leading to malignant transformation. Figure A shows primary renal cell carcinoma histology with rounded or polygonal clear cells with abundant clear cytoplasm. Incorrect Answers: 2-5. Renal cell carcinoma is a malignancy of proximal tubular epithelial cells; all other cell lines mentioned are not consistent with renal cell carcinoma. For Step exams it is important to know the epidemiology surrounding RCC. RCC is one of the 10 most common forms of cancer in both men and women (1). 90% of these cancers are believed to be of epithelial cell origin and are referred to as RCC.

A 67-year-old man presents to the clinic for bloody urine for 1 day. He denies any recent trauma, medication changes, abnormal ingestion, discharge, or pain associated with his symptoms. He endorses increased urinary frequency and urgency within the past week. His past medical history is significant for diabetes, asthma, and a past parasitic infection when he visited Egypt 10 years ago. He also had an episode of idiopathic uveitis that was resistant to standard therapy and was subsequently treated with a 2-week-course of cyclophosphamide. A urinalysis is unremarkable. A cystoscopy is performed and its finding is shown in Figure A. Which of the following most likely led to this patient's condition? A. Benign prostatic hyperplasia B. Cyclophosphamide exposure C. Repeated urinary tract infections D. Prolonged exercise E. Schistosoma haematobium infection

This patient has squamous cell carcinoma of the bladder (e.g., painless gross hematuria and positive cystoscopy findings demonstrating squamous cell infiltration), which is likely due to his past Schistosoma haematobium infection (e.g., parasitic infection when he visited Egypt). Squamous cell carcinoma of the bladder accounts for a small percentage of bladder cancers in North America but is seen in up to 75% of bladder cancers in areas where S. haematobium infection is endemic (Africa and the Middle East). Other risk factors include chronic or recurrent urinary tract infections (UTIs), bladder calculus, pelvic radiation therapy, prolonged exposure to cyclophosphamide (especially with hemorrhagic cystitis), and smoking. Patients often present with painless hematuria. Diagnosis is confirmed via cystoscopy with biopsy. Figure/Illustration A is a pathologic slide of the bladder demonstrating squamous cell carcinoma with characteristic keratin pearls (arrows). Incorrect Answers: Answer 1: Benign prostatic hyperplasia (BPH) is a common finding in elderly men and may present as microscopic hematuria but rarely causes gross hematuria. Although this patient is complaining of urinary symptoms suggestive of BPH, increased urinary frequency and urgency are also a result of bladder irritation secondary to malignancy. Answer 2: Cyclophosphamide exposure is one of the risk factors of transitional cell carcinoma of the bladder. However, prolonged exposure of drug, especially in cases with hemorrhagic cystitis, is associated with the development of malignancy. This patient only had a short course of treatment with cyclophosphamide. Answer 3: Repeated UTI is a risk factor associated with the disease. However, this patient's past medical history does not suggest repeated UTIs. Answer 4: Prolonged exercise can lead to exercise-induced hematuria and is often seen in long-distance runners. This patient is not reporting such activities. Bullet Summary: Schistosoma haematobium infection is a strong risk factor for the development of squamous cell carcinoma of the bladder.

A 72-year-old male presents to his primary care physician with urinary hesitancy and urinary dribbling that began 6 weeks ago and has gradually worsened. Rectal exam reveals a markedly enlarged prostate. CT scan, shown in Figure A, demonstrates dilated ureters and renal pelvises. Which of the following likely accounts for the CT scan results A. Urethral obstruction B. Autosomal dominant polycystic kidney disease C. Prerenal azotemia D. Elevated serum PSA E. Multiple endocrine neoplasia type 1

The patient's clinical scenario suggests urethral obstruction due to prostatic hyperplasia. Prolonged obstruction can lead to hydronephrosis as seen in the patient's CT scan. Urinary hesitancy, urinary intermittency, straining to void, and urinary dribbling are symptoms associated with urethral obstruction due to prostatic hyperplasia, which can be felt on rectal examination in about half of all cases. Prolonged urinary obstruction can cause unilateral or bilateral hydronephrosis. Hydronephrosis can be acute or chronic and is marked by a dilated ureter and/or renal pelvis. Edwards reviews benign prostatic hyperplasia (BPH). Symptoms include urinary hesitancy, weak stream, nocturia, incontinence, and recurrent urinary tract infections. Acute urinary retention is rare and requires bladder catheterization. Irreversible renal damage due to urinary obstruction is possible but rare.Sutaria et al. review hydronephrosis in the elderly due to urinary tract obstruction. Lower urinary tract obstruction should always be evaluated in individuals with declining kidney function. Removing any present obstruction can rapidly reverse renal decline, while chronic obstruction can produce permanent kidney damage. Figure A is a CT scan showing grossly dilated ureters due to hydronephrosis. Incorrect Answers: Answer 2: Hypertension, hematuria, and abdominal discomfort are symptoms of autosomal dominant polycystic kidney disease. Answer 3: Hypoperfusion of the kidneys is responsible for prerenal azotemia. Hemorrhage, shock, volume depletion, congestive heart failure, and narrowing of the renal artery are common causes of this condition. Answer 4: Prostate cancer and benign prostatic hyperplasia (BPH) may cause elevated serum PSA. Elevation in serum PSA, however, is not a direct cause of hydronephrosis. Answer 5: Multiple endocrine neoplasia type I (MEN I) refers to a group of linked tumors in the pituitary, pancreas, and parathyroid glands. None of these conditions produces hydronephrosis. Note Highlight (default)

A 45-year-old man presents to his primary care provider for his yearly physical. On exam, the physician discovers a hard, testicular mass on the right testis. On further questioning, the patient remarks that he first noticed it 6 months ago, but did not think he needed to have it examined since it wasn't painful. Transillumination of the scrotum does not reveal translucency. The ultrasound of his right testis is shown in Figure A. Blood work reveals normal AFP and Beta-hCG. Chest radiograph and CT scan do not show positive lymph nodes or distant metastases. Which of the following is the most appropriate next step? A. Biopsy B. Radical orchiectomy C. Chemotherapy D. Three month re-imaging E. No further workup, instruct patient to return if symptomatic

The patient's presentation and workup is consistent with testicular cancer, despite the normal blood work. All testicular cancers should undergo radical orchiectomy. Testicular cancer is the most common malignancy in males of age 15-35, the vast majority of which are germ cell tumors. They typically present as a non-acute, painless testicular mass, although some patients do report discomfort. Other manifestations may include gynecomastia, paraneoplastic hyperthyroidism, and paraneoplastic limbic encephalitis. Workup of a hard testicular mass includes ultrasound, serum tumor markers, and CT scan. Once diagnosed, the first step is radical inguinal orchiectomy with possible retroperitoneal lymph node dissection to evaluate for metastases. Following surgery and staging, chemotherapy, radiation therapy, or active surveillance are all viable options. Overall survival rates for testicular cancer are quite high. Tiemstra and Kapoor discuss the evaluation of scrotal masses, and present an algorithm (Illustration A) for appropriate diagnosis. They emphasize a complete understanding of the anatomy and being able to identify the source of the pathology. Doppler ultrasonography is also mentioned as an important diagnostic tool in the evaluation of testicular pathology. Groll et al. performed a literature review on the role of testicular germ cell tumor surveillance following surgery as opposed to adjuvant therapies. Because the outcomes for early stage testicular cancer are so good, they take into consideration other factors such as quality of life, impact on sex and fertility, and compliance. They conclude that surveillance is a preferred approach in stage I seminoma and non-seminomatous germ cell tumors, although they recommend that further research is conducted to explore other psychosocial aspects of post-surgical management. Figure A is an ultrasound of the right testis revealing a well-defined hypoechoic lesion without cystic areas. Illustration A is an algorithm for diagnosing scrotal masses. Incorrect answers: Answer 1: Biopsy is very rarely indicated in testicular cancer. Answer 3: Chemotherapy is generally used as an adjuvant therapy following radical orchiectomy. Answer 4: This presentation is consistent with testicular cancer. Delay in surgery would not be appropriate. Answer 5: Testicular cancer is often painless, and requires treatment.

A 12-year-old boy presents with right flank pain and hematuria. His older brother has had several similar episodes in the past. Results of microscopic urinalysis are shown in Figure A. Which of the following tests can be used to help confirm the underlying diagnosis? A. Cyanide-nitroprusside test B. Serum uric acid concentration C. Urine uric acid concentration D. Urine culture E. Urease test

This boy has the classic presentation of kidney stones, a positive family history, and microscopic analysis showing hexagonal-shaped crystals. These findings are suggestive of cystinuria, which can be confirmed with the cyanide-nitroprusside test. Kidney stones classically present with abrupt-onset renal colic and severe flank pain, which usually radiates toward the abdomen and groin. Patients often have dysuria and hematuria. Kidney stones are most often seen in adults in their 30's-40's. When kidney stones are seen in children, hereditary predisposing factors should be considered. Kidney stones can be of several types, including calcium stones, struvite stones, uric acid stones, and cystine stones. Goldfarb et al. discuss the different types of kidney stones. Nearly 80% of stones are calcium stones, with the majority of these being calcium oxalate stones. 10% of stones are uric acid stones, and these are more frequently seen in patients with a history of gout. An additional 10% are struvite stones, which are caused by infection with urease-producing bacteria. Finally, cystine stones make up around 1% of all stones. Cystine stones are caused by an autosomal recessive disorder; a strong family history of stones in childhood increases the probability of cystine stones. Byrd et al. review the use of the cyanide-nitroprusside test (CNT) in the diagnosis of cystinuria. The authors performed the CNT on urine samples from 43 patients with a pre-existing confirmed diagnosis of cystinuria. In 95% of cases, the CNT correctly diagnosed cystinuria. The authors concluded that the CNT is an inexepensive and reliable test for the diagnosis of cystinuria. Figure A shows hexagonal crystals classically seen in cystinuria and cystine stones. Illustration A shows the classic microscopic appearance of struvite stones. Incorrect Answers: Answers 2,3: Serum or urine uric acid levels may be appropriate in the workup for a patient with uric acid stones. However, the hexagonal-shaped stones seen on microscopy are highly characteristic of cystine stones. Answer 4: Urine culture would be an appropriate step in the workup of a suspected infection. Infection with urease-producing organisms can lead to struvite stones. Answer 5: Again, infection with urease-producing organisms can lead to struvite stones.

A 67-year-old man presents to his primary care physician complaining of frequent urination overnight. He states that for several years he has had trouble maintaining his urine stream along with the need for frequent urination, but the nighttime urination has only recently started. The patient also states that he has had 2 urinary tract infections in the last year, which he had never had previously. On exam, his temperature is 98.8°F (37.1°C), blood pressure is 124/68 mmHg, pulse is 58/min, and respirations are 13/min. On digital rectal exam, the prostate is enlarged but feels symmetric and smooth. Which of the following is a possible consequence of this condition? A. Increased serum AFP B. Increased serum ALP C. Increased serum creatinine D.Increased serum hCG E. Malignant transformation

This patient has classic findings of benign prostatic hyperplasia (BPH), which can result in acute urinary retention. Over time, urinary retention can lead to a decline in renal function and increased serum creatinine. BPH is common in elderly men and is characterized by hyperplasia of the middle and lateral lobes of the prostate. In comparison, prostatic adenocarcinoma is more often seen in the posterior lobe. The common symptoms of BPH include increased urinary frequency, dysuria, difficulty maintaining the urinary stream, and nocturia. Because of the urinary retention, patients can develop recurrent urinary tract infections and hydronephrosis. Incorrect Answers: Answer 1: Increased serum AFP is seen with yolk sac (endodermal sinus) tumors, which are more common in young boys. Answer 2: Increased serum ALP can be seen with prostatic adenocarcinoma, due to bony metastases that arise from osteoblastic activation. Answer 4: Increased serum hCG is found in testicular germ cell tumors such as choriocarcinoma, teratoma, and embryonal carcinoma. Answer 5: Malignant transformation results in prostatic adenocarcinoma; however, BPH is not premalignant. Bullet Summary: Benign prostatic hyperplasia is hyperplasia of the middle and lateral lobes of the prostate, leading to dysuria, increased frequency, and nocturia; long-term effects include urinary retention with hydronephrosis and decreased renal function.

A 68-year-old man presents to his primary care provider after noticing that his urine has been pink for the last week. He does not have any pain with urination, nor has he had any associated fevers or infections. On his review of systems, the patient notes that he thinks he has lost some weight since his belt is looser, and he has also had occasional dull pressure in his back for the past two months. His temperature is 98.8°F (37.1°C), blood pressure is 132/90 mmHg, pulse is 64/min, and respirations are 12/min. The patient weighs 210 lbs (95.3 kg, BMI 31.9 kg/m^2), compared to his weight of 228 lbs (103.4 kg, BMI 34.7 kg/m^2) at his last visit 2 years prior. On exam, the patient does not have any back or costovertebral angle tenderness. On abdominal palpation, a firm mass can be appreciated deep in the left abdomen. The clinical workup should also assess for which of the following conditions? A. Carcinoid syndrome B. Cushing syndrome C. Lambert-Eaton myasthenic syndrome D. Myelitis E. Syndrome of inappropriate antidiuretic hormone secretion (SIADH)

This patient likely has renal cell carcinoma (RCC) due to his hematuria, weight loss, dull pressure, and palpable mass. RCC is associated with numerous paraneoplastic syndromes, including Cushing syndrome. RCC most commonly affects elderly men and commonly presents with hematuria. Fewer than half of cases are associated with back pain, which may also present as pressure instead of pain. A mass can be palpated over time as well and can be confirmed on imaging. Weight loss and recurrent fevers can also be seen with RCC. A biopsy is needed to confirm the diagnosis. RCC produces several peptides, such as EPO, ACTH, and PTHrP, and is thus associated with paraneoplastic conditions such as polycythemia, Cushing syndrome, and hypercalcemia. Incorrect Answers: Answer 1: Carcinoid syndrome is associated with neuroendocrine tumors, especially of the small bowel. The symptoms of flushing and diarrhea are due to the secretion of compounds such as serotonin and kallikrein. Answer 3: Lambert-Eaton myasthenic syndrome can be associated with small cell carcinoma of the lung. This tumor can produce antibodies against presynaptic calcium ion channels. Answer 4. Paraneoplastic myelitis can also occur with small cell carcinoma of the lung, when the tumor produces antibodies against neurons. Answer 5: SIADH can be paraneoplastic when ADH is produced from small cell lung cancer or from intracranial neoplasms. Bullet Summary: Renal cell carcinoma can produce ACTH, leading to paraneoplastic Cushing syndrome; additionally, renal cell carcinoma can cause polycythemia and hypercalcemia.

A 49-year-old African American female with a history of chronic myeloid leukemia for which she is receiving chemotherapy presents to the emergency room with oliguria and colicky left flank pain. Her serum creatinine is 3.3 mg/dL. What is the preferred preventative therapy that could have been administered to this patient to prevent her complication of chemotherapy? A. Diuresis B. Acidification of the urine C. Colchicine D. Steroids E. Dialysis

This patient most likely has acute kidney injury (AKI) and a renal stone secondary to tumor lysis syndrome (TLS). The chief TLS prevention is hydration and diuresis to increase urine flow rate. TLS is a condition following chemotherapy treatment which causes tumor cells to lyse and release uric acid, a metabolite of tumor nucleic acid. Importantly, the best management of TLS is prevention, as opposed to treating once symptoms begin. Note that uric acid stones require an acidic environment to precipitate, while magnesium ammonium phosphate (struvite) stones require an alkaline environment to precipitate. Frassetto et al. review risk factors and prevention of kidney stones, which are increasingly prevalent among women and the elderly. Obesity, medications such as protease inhibitors and diuretics, high-animal protein diets, and dehydration all promote stone formation. The authors advocate vegetable-rich diets and supplemental citrate to prevent uric acid, calcium oxalate, and cystine stones. Coiffier et al. propose new guidelines in the prevention and management of patients at risk of developing TLS. Prevention strategies include hydration, prophylactic rasburicase, and allopurinol. The aggressiveness of treatment depends on the individual patient's degree of TLS risk. Of note, they do not recommend urine alkalinization as a preventative measure. Incorrect Answers: Answer 2: Urine alkalinization, not acidification, has been proposed as a preventative measure for TLS but does not have a large body of supporting evidence. Answer 3: Allopurinol, not colchicine, has been used in the prevention of TLS. Answer 4: Steroids do not have a role in the prevention of TLS. Answer 5: While dialysis may be used in extreme cases of AKI secondary to TLS, it is not the preferred preventative therapy.

A 72-year-old man with a 35 pack-year smoking history presents to his primary care physician with a 3 day history of fever. He says that he has been feeling tired and lethargic over the last few months but finally came in after he developed the fever. He has also noticed that his urine has been darker than usual over the same time period. Physical exam reveals costovertebral angle tenderness and a palpable mass in the left lower quadrant. He has also lost 15 pounds since the last visit a year ago despite no change in his lifestyle. Based on clinical suspicion a panel of laboratory tests are obtained. Which of the sets of results seen in Figure A would most likely be seen in this patient? A B C D E You need to see picture for it to make sense google this!!!!!

This patient who has fever, weight loss, probable hematuria, costovertebral angle tenderness, and a palpable abdominal mass most likely has renal cell carcinoma. Patients with this disease will have hypertension and hypercalcemia due to paraneoplastic syndromes. Renal cell carcinoma is a malignancy of proximal renal tubular cells that most commonly affects male smokers. Since this area of the kidney has many functions, renal cell carcinoma has a large number of paraneoplastic syndromes associated with the disease. For example, the release of erythropoietin by the tumor often leads to polycythemia with increased red blood cell production despite ongoing hematuria. Similarly, increased production of renin may lead to hypertension due to retention of sodium. Patients may also present with hypercalcemia due to ectopic release of parathyroid hormone-related protein. Finally, patients can have hypercortisolism due to ectopic production of adrenocorticotropic hormone. Answer 1: Decreased blood pressure, hypercalcemia, and normal cortisol is incorrect because overproduction of renin will result in increased blood pressure. Answer 2: Hypertension, hypocalcemia, and normal cortisol is incorrect because renal cell carcinoma can release parathyroid hormone-related protein, which subsequently increases serum calcium levels. Answer 3: Hypertension, hypocalcemia, and hypocortisolism is incorrect because cortisol will either be normal or increased if there is production of adrenocorticotropic hormone. Furthermore, calcium will either be normal or increased if there is release of parathyroid hormone-related protein. Answer 5: Hypertension, hypercalcemia, and hypocortisolism is incorrect because cortisol will either be normal or increased if there is production of adrenocorticotropic hormone. Bullet Summary: Renal cell carcinoma can present with paraneoplastic hypertension, hypercalcemia, hypercortisolism, and polycythemia.

A 51-year-old woman presents to her primary care physician with a 6-month history of urinary problems. Specifically, she says that she has had several embarrassing episodes where she urinated during a meeting at work. She says that she felt the urge to urinate but she was not able to make it to the bathroom in time. She also has experienced nighttime urges to urinate that disrupt her sleep. About one-third of these episodes she says that starts urinating in bed before she is able to start going to the bathroom. Her past medical history is significant for diabetes and fibroids. She has 3 living children all of whom were delivered vaginally. Which of the following mechanisms is most likely responsible for this patient's symptoms? A. Bladder outlet obstruction B. Decreased sphincter function C. Detrusor muscle overactivity D. Detrusor muscle underactivity E. Weakened pelvic floor muscles

This patient who is experiencing an inability to reach the bathroom in time after feeling the urge to urinate most likely has urge incontinence, which is caused by overstimulation of the detrusor muscle. Urinary incontinence can be divided into stress, urge, and overflow incontinence. Urge incontinence occurs when the detrusor muscle becomes overstimulated. It presents with an inability to reach the bathroom in time after feeling the urge to urinate. Notably, the detrusor muscle is innervated by pelvic splanchnic nerves of the parasympathetic division of the autonomic nervous system. These nerves release acetylcholine, which causes contraction of the bladder by binding to muscarinic receptors. Therefore, the classic treatment for urge incontinence is oxybutynin, which works by inhibiting these receptors. Incorrect Answers: Answer 1: Bladder outlet obstruction may lead to incontinence; however, this obstruction will result in overflow rather than urge incontinence. Answer 2: Decreased sphincter function may lead to incontinence; however, sphincter malfunction will result in stress rather than urge incontinence. Answer 4: Detrusor muscle underactivity may lead to incontinence; however, underactivity will result in incomplete emptying and overflow rather than urge incontinence. Answer 5: Weakened pelvic floor muscles may lead to incontinence; however, weakened floor muscles will result in stress rather than urge incontinence. Bullet Summary: Urge incontinence is caused by detrusor muscle overactivity and results in urinary leakage at night despite feeling the urge to urinate.

An 82-year-old man presents to his primary care physician with a 1-month history of increasing nocturnal urination. Specifically, he states that he has been waking up 3-4 times per night to urinate. His past medical history is significant for hypertension and diabetes. Based on these symptoms he is started on a medication and told that he should avoid medications for erectile dysfunction. Which of the following changes would most likely be observed in this patient's cells after he is started on this medication? A. Decreased protein kinase A activity B. Decreased protein kinase C activity C. Decreased 5-alpha reductase activity D. Increased protein kinase A activity E Increased protein kinase C activity

This patient who presents with increasing nocturnal urination and who was told to avoid erectile dysfunction drugs was most likely started on tamsulosin (which decreases protein kinase C activity) for benign prostatic hyperplasia. The key is that alpha-1-adrenergic antagonists can cause severe hypotension when combined with phosphodiesterase-5 inhibitors. Benign prostatic hyperplasia causes expansion of the prostate that results in lower urinary tract symptoms such as increased urinary frequency, nocturia, and weak urinary stream. It presents primarily in older men and is associated with obesity, hypertension, and diabetes. Medical treatment of this disease includes alpha-1-adrenergic antagonists such as terazosin and tamsulosin as well as 5-alpha-reductase inhibitors such as finasteride. The alpha-1-adrenergic receptor is linked to the G-alpha-q second messenger pathway, so inhibition of this receptor will lead to decreased protein kinase C activity. Of note, these drugs should not be combined with phosphodiesterase-5 inhibitors because they can synergistically cause severe hypotension. Incorrect Answers: Answer 1: Decreased protein kinase A activity would be seen with inhibition of the G-alpha-s pathway; however, this pathway is linked to beta-1-adrenergic and beta-2-adrenergic receptors rather than alpha-1-adrenergic receptors. Beta-blockers would be an example of a drug class that has this effect on cells. Answer 3: Decreased 5-alpha reductase activity describes the mechanism of action for finasteride, which can also be used in the treatment of benign prostatic hyperplasia; however, this drug does not have a significant interaction with phosphodiesterase-5 inhibitors. Finasteride works by decreasing dihydrotestosterone and thereby shrinking the enlarged prostate. Answer 4: Increased protein kinase A activity would be seen with activation of the G-alpha-s pathway, which would be seen in drugs that activate the beta-2-adrenergic pathway. Albuterol would be an example of a drug that has this effect on cells. Answer 5: Increased protein kinase C activity would be seen in activation of the G-alpha-q pathway; however, tamsulosin is an antagonist of the alpha-1-adrenergic pathway rather than an agonist. Phenylephrine would be an example of a drug that has this effect on cells. Bullet Summary: Benign prostatic hyperplasia can be treated with alpha-1-adrenergic antagonists such as tamsulosin.

A 55-year-old man is brought to the emergency department by ambulance after being found disoriented. He has limited ability to communicate in English but indicates that he has left flank pain and a fever. Chart review reveals that he has diabetes and sleep apnea but both are well controlled. He also has a 30-pack-year smoking history and has lost about 20 pounds since his last presentation. Physical exam reveals a bulge in his left scrotum and ultrasound reveals bilateral kidney stones. Which of the following findings is also associated with the most likely cause of this patient's symptoms? A. Aniridia B. Cavitary lung lesion C. Gynecomastia D. Increased hematocrit E. Jaundice

This patient with a smoking history who presents with flank pain, fever, left varicocele and signs of hypercalcemia most likely has renal cell carcinoma, which is also associated with polycythemia due to production of erythropoietin. Renal cell carcinoma is a tumor of proximal renal tubular cells that usually presents with fever, flank pain, weight loss, and hematuria. In addition, it is associated with smoking because the kidney is responsible for filtering smoking related toxins. Similarly, a left-sided tumor can block the testicular vein and cause a left-sided varicocele. It is important to understand that renal cell carcinoma can be associated with several paraneoplastic syndromes due to ectopic production of hormones. These include hypertension due to renin production, polycythemia due to erythropoietin production, hypercalcemia (renal stones and confusion) due to parathyroid hormone-related protein, and hypercortisolism due to production of adrenocorticotropic hormone. Incorrect Answers: Answer 1: Aniridia can be seen in syndromic association with Wilms tumor; however, this disease presents in children usually younger than 5 years of age. Answer 2: Cavitary lung lesions are associated with squamous cell carcinoma of the lung; however, this disease does not present with flank pain or with a left-sided varicocele. Answer 3: Gynecomastia can be seen in testicular tumors; however, the constellation of symptoms seen in this case is more consistent with renal cell carcinoma with a left-sided varicocele due to obstruction of the testicular vein. Answer 5: Jaundice is associated with cholangiocarcinoma, which presents with fever and pain but also includes symptoms such as pruritis, dark urine, and clay-colored stool. Bullet Summary: Renal cell carcinoma is associated with multiple paraneoplastic syndromes such as polycythemia due to production of erythropoietin.

A 35-year-old woman presents to her primary care provider reporting intermittent pain in her left side that is worse with urination. She states that she has also been urinating more frequently and is waking up at night to urinate. She was previously healthy, although she has had two episodes of urinary tract infections that resolved with antibiotics. Her temperature is 98.4°F (36.9°C), blood pressure is 110/78 mmHg, pulse is 65/min, and respirations are 12/min. The patient does not have any tenderness on exam, and genitourinary exam is grossly normal. A urinalysis is performed, demonstrating a urine pH of 7.2, negative leukocyte esterase, and negative bacteria. Large quantities of crystals are seen, as shown in Figure A. Which of the following is the likely composition of these crystals? A. Ammonium magnesium phosphate B. Calcium oxalate C. Calcium phosphate D. Cystine E. Urate

This patient with alkaline urine containing coffin lid-shaped crystals has a struvite stone composed of ammonium magnesium phosphate. Ammonium magnesium phosphate crystals, also known as struvite, are a type of urinary system stone, responsible for approximately 10-15% of kidney stones. Struvite is formed in patients with a history of bacterial infection from organisms that are capable of hydrolyzing urea to ammonium, including Proteus mirabilis, Staphylococcus saprophyticus, Pseudomonas, and Klebsiella. Struvite stones tend to be found more commonly in women than in men, and almost always are preceded a history of urinary tract infection. Struvite precipitation is potentiated by an alkaline urinary pH, and can be seen as coffin lid-shaped crystals in urinary sediment. Large struvite stones can be seen as radiopaque staghorn calculi on imaging. Treatment involves antibiotics to treat the underlying infection and inhibit stone growth, and removal of the stone with shock wave lithotripsy or percutaneous nephrolithotomy. In patients with recurrent struvite stones, chronic treatment has been attempted with the bacterial urease inhibitor acetohydroxamic acid and prophylatic antibiotics. Figure A shows coffin lid-shaped struvite crystals composed of ammonium magnesium phosphate. Incorrect Answers: Answer 2: Calcium oxalate stones are radiopaque, envelope- or dumbbell-shaped stones that precipitate at decreased urinary pH. Calcium oxalate crystals can form in various pathologies that result in hypercalciuria, or from conditions such as ethylene glycol ingestion or vitamin C overdose. Answer 3: Calcium phosphate stones precipitate in alkaline urine and can convert to hydroxyapatite, which forms larger stones than calcium oxalate. These stones are also far more common than calcium oxalate stones. Calcium phosphate stones appear as wedge-shaped prisms. Answer 4: Cystine stones precipitate in acidic urine and are hexagonal. These stones form due to impaired cystine reabsorption in the proximal convoluted tubule and can also form staghorn calculi. The cyanide-nitroprusside test is used for diagnosis. Answer 5: Urate crystals precipitate in acidic urine and are associated with hyperuricemia or diseases with increased cell turnover. Crystals are rhomboid and radiolucent. Bullet Summary: Struvite stones are composed of coffin lid-shaped ammonium magnesium phosphate crystals.

A 28-year-old woman presents to her primary case physician because she has been experiencing severe left flank pain. She says that the pain comes in squeezing, spams like bursts and that it progresses from her left flank to her left groin. On physical exam, there is left-sided costovertebral angle tenderness. A urine sample is obtained for laboratory testing. Microscopy findings from the urine sample are shown in Figure A. Which of the following is associated with the most likely cause of this patient's symptoms? A. Antifreeze abuse B. Decreased amino acid reabsorption C. Gout D. Infection E. Renal tubular acidosis

This patient with colicky flank pain, CVA tenderness, and "coffin-lid" crystals on urine microscopy most likely has struvite crystal nephrolithiasis secondary to infection. Nephrolithiasis is a disease caused by the development of stones in the urinary system either involving the kidney or the ureters. These stones cause obstruction and irritation of the surrounding structures and lead to colicky flank pain. Importantly, the stones seen in this disease can be composed of a variety of substances depending on the etiology underlying the disease. Specifically, struvite crystals are composed of magnesium ammonium phosphate and are created by urease-positive microorganisms such as Proteus species. These crystals are classically "coffin-lid" shaped in appearance. Figure A shows urine microscopy findings of coffin-lid shaped crystals, which are classically seen in struvite nephrolithiasis. Incorrect Answers: Answer 1: Antifreeze abuse can cause calcium oxalate stones; however, these crystals have a classic envelope or dumbbell shape. Answer 2: Decreased amino acid reabsorption is associated with the development of cystine stones; however, these crystals have a classic hexagonal shape. Answer 3: Gout is associated with the development of uric acid stones; however, these crystals have a classic rhomboid or rosette shape. Answer 5: Renal tubular acidosis can lead to the development of calcium phosphate stones; however, these crystals have a classic wedge-shaped prism shape. Bullet Summary: Urease-positive organisms can lead to the production of struvite stones with a coffin-lid shape.

A 68-year-old man presents to his primary care physician with a 4-week history of back pain. He says that the pain does not appear to be associated with activity and is somewhat relieved by taking an NSAID. Furthermore, he says that he has had increasing difficulty trying to urinate. His past medical history is significant for kidney stones and a 30-pack-year smoking history. Radiographs reveal osteoblastic lesions in the spine. Which of the following drugs would most likely be effective in treating this patient's disease? A. Continuous leuprolide B. Imatinib C. Pulsatile leuprolide D. Rituximab E. Tamsulosin

This patient with osteoblastic back lesions and difficulty urinating most likely has prostate adenocarcinoma, which can be treated by continuous administration of leuprolide. Prostatic adenocarcinoma is the most common cancer in men and usually arises from the posterior lobe of the prostate. Given that this region is far from the urethra, this disease can develop asymptomatically; however, it can cause obstructive uropathy if it invades the neck of the bladder. It can also present with low back pain if it metastasizes to the spine where it will form osteoblastic lesions that can be seen on radiography. Notably, since prostatic adenocarcinoma arises from a hormone responsive tissue, continuous administration of the gonadotropin-releasing hormone analogue leuprolide can suppress testosterone production and tumor cell proliferation. Incorrect Answers: Answer 2: Imatinib is a tyrosine kinase inhibitor that can be used to treat chronic myeloid leukemia; however, this disease would not present with obstructive uropathy. Answer 3: Pulsatile leuprolide would stimulate production of sex steroids and can be used to treat infertility; however, this is the opposite of how this drug is used for prostate adenocarcinoma. Answer 4: Rituximab is an anti-CD20 monoclonal antibody that can be used to treat a variety of lymphomas; however, these diseases would not present with both obstructive uropathy and back pain. Answer 5: Tamsulosin is an alpha-antagonist that is used for benign prostatic hyperplasia; however, it would not be used to treat prostate adenocarcinoma. Bullet Summary: Prostatic adenocarcinoma can be treated by continuous administration of the gonadotropin-releasing hormone analogue leuprolide.

A 32-year-old man presents to his primary care physician complaining of pain accompanied by a feeling of heaviness in his scrotum. He is otherwise healthy except for a broken arm he obtained while skiing several years ago. Physical exam reveals an enlarged "bag of worms" upon palpation of the painful scrotal region. Shining a light over this area shows that the scrotum does not transilluminate. Which of the following statements is true about the most likely cause of this patient's symptoms? A. Equally common on both sides B. More common on left due to drainage into inferior vena cava C. More common on left due to drainage into renal vein D. More common on right due to drainage into inferior vena cava E. More common on right due to drainage into renal vein

This patient with scrotal pain and fullness with a "bag of worms" on palpation most likely has a varicocele, which is more common on the left because the left gonadal vein drains into the left renal vein. Varicoceles are produced by increased scrotal venous pressure causing dilated veins in the pampiniform plexus. Varicoceles present with scrotal pain and fullness, a "bag of worms" on palpation, and a scrotum that does not transilluminate. It is most often seen on the left side because of the different drainage patterns of the two gonadal veins. Whereas the right gonadal vein drains directly to the inferior vena cava, the left side experiences increased resistance because it drains into the left renal vein. Incorrect Answers: Answer 1: Varicocele is not equally common on both sides, rather it is more common on the left side. Answer 2: Varicocele is more common on the left side but this is because it drains into the renal vein rather than the inferior vena cava. Answer 4: Varicocele is not more common on the right side, rather it is more common on the left side. Answer 5: Varicocele is not more common on the right side, which drains into the inferior vena cava rather than the renal vein. Bullet Summary: Varicocele is more common on the left because the left gonadal vein drains into the left renal vein rather than the inferior vena cava.

A 54-year-old male presents to clinic complaining that he is not sleeping well because he has to get up from bed to urinate multiple times throughout the night. He says that he strains to void, has terminal dribbling, and has urinary urgency. Past medical history is significant for orthostatic hypotension. On digital rectal exam, you note symmetric firm enlargement of the prostate. Free Prostate-Specific-Antigen (PSA) level is 4.6 ng/mL. Before you finish your physical exam, the patient asks if there is anything you can do for his male-pattern baldness. What is the mechanism of action of the drug that would pharmacologically treat this patient's urinary issues and his male-pattern baldness? A. Phosphodiesterase-5 inhibition B. Alpha-1 blockade C. Squalene epoxidase inhibition D. 5-alpha reductase inhibition E. 17,20-desmolase inhibition

This patient's nocturia, terminal dribbling, urinary urgency, symmetrically enlarged prostate and elevated PSA levels suggest benign prostatic hyperplasia (BPH). His BPH symptoms, history of orthostatic hypotension, and male pattern baldness makes a 5-alpha reductase inhibitor appropriate. BPH is very common among older men. It mainly develops in the transitional and periurethral zones of the prostate, leading to nodule formation that can compress the urethra. Pathogenesis is not completely understood; however, androgen-dependent stromal and glandular cell growth may play a major role. Dihydrotestosterone (DHT) is produced from testosterone via the enzyme 5-alpha reductase. DHT binds to nuclear androgen receptors that ultimately lead to prostatic epithelial and stromal cell growth. DHT is also responsible for male pattern baldness. Enzyme inhibition by finasteride is a reasonable pharmacologic agent in patients who can't tolerate alpha-1 blockade. It is also used for male pattern baldness. Edwards explores the diagnosis and management of BPH. Frequency, severity of symptoms, and quality of life should all be elicited as part of the initial history, and a digital rectal exam and urinalysis should be performed in patient suspsected of having BPN. Alpha blockers provide relatively rapid improvement of symptoms. 5-alpha reductase inhibitors may decrease prostate size and alter the long-term clinical course, even though their effects takes longer (approximately 6 months) to improve patient symptoms. Bartsch et al. discusses the use of 5-alpha reductase inhibition in BPH. Finasteride has been used to treat both BPH and male-pattern baldness. It is most effective in patients with large prostates, according to their meta-analysis. Finasteride as a long-term pharmocologic therapy can alter clinically significant endpoints, such as acute urinary retention or the requirement of surgical correction. Illustration A shows the zones of the prostate. Incorrect Answers: Answer 1: Phosphodiesterase-5 (PDE-5) inhibitors, such as sildenafil, increase cyclic guanosine monophosphate levels, causing vascular smooth muscle relaxation. This drug can be used for erectile dysfunction and pulmonary hypertension. Answer 2: Alpha-1 blockers are the most commonly prescribed medication for BPH. They relax the smooth muscle sphincters of the urethra. Orthostatic hypotension and dizziness are the most important side effects. Selective prostatic alpha-receptor blockers (tamsulosin) have little to no effect on blood pressure; however, it would not treat the patient's male patterned baldness. Answer 3: Terbinafine is a squalene epoxidase inhibitors that is a first-line therapeutic (along with griseofulvin) agent in the treatment of tinea capitis in children. Answer 5: Ketoconazole is a an antifungal that inhibits cytochrome P450 (17,20 desmolase), which is necessary for steroid synthesis.

A 6-month-old male presents with a painless, enlarged left scrotum. After examining the patient, you suspect this enlargement is secondary to serous fluid entering and accumulating in the scrotum through a patent processus vaginalis. Which of the following would be the most useful next step in confirming the diagnosis of this patient's condition? A. Measurement of AFP and hCG levels B. Transillumination test followed by scrotal ultrasound C. Evaluation of cremasteric reflex on physical exam D. Measurement of serum testosterone levels E. Standard urinalysis

This patient's presentation is consistent with a left-sided hydrocele. The diagnosis of hydrocele is confirmed through a positive illumination test and a scrotal ultrasound, which can differentiate solid masses from fluid. Hydrocele often occurs due to failed obliteration of the processus vaginalis, the communication between the tunica vaginalis and the peritoneum. Fluid, such as serous fluid, blood (hematocele), or sperm (spermatocele), accumulates in the tunica vaginalis of the scrotum, resulting in the enlarged appearance. Hydrocele is often associated with indirect inguinal hernia. Surgery is indicated to close the patent processus vaginalis. Galejs discusses the diagnosis and management of scrotal abnormalities. In evaluating such patients, physical examination should include inspection and palpation of the abdomen, testes, epididymis, scrotum, and inguinal region. Urinalysis should routinely be performed; however, scrotal imaging such as ultrasound should only be pursued when the diagnosis remains unclear after initial evaluation. Lao et al. address the management of hydrocele in very young patients, concluding that hydroceles are relatively common in infants and often resolve without operative intervention. As a general rule, hydroceles should be expected to resolve by 1 year of age. After this time period, surgery should be pursued to close a persistent patent processus vaginalis and thereby decrease the risk of future complications such as inguinal hernias. Illustration A depicts a positive transillumination test demonstrating the presence of hydrocele. Illustration B shows the appearance of hydrocele on ultrasound. Incorrect Answers: Answer 1: These tumor markers would be useful in the diagnosis of a testicular tumor; generally one or both would be expected to be increased, depending on the type of tumor present. Answer 3: This finding is consistent with testicular torsion, which would present with a sudden onset of acute testicular pain. Answer 4: Serum testosterone levels would not be expected to be disturbed by a hydrocele. Answer 5: No abnormalities would be anticipated on urinalysis, therefore, it is unlikely to be helpful.

A 27-year-old male presents with primary complaints of a palpable mass in his scrotum and mild testicular pain. Physical exam reveals an abnormal appearing scrotum around the left testis, as depicted in image A. Which of the following is the most likely etiology of this presentation? A. Compression of the left renal vein at the aortic origin of the superior mesenteric artery B. Patent processus vaginalis allowing fluid entry into the scrotum C. Neisseria gonorrhoeae Infection of the left testis leading to epididymitis D. Unilateral failure of the left testis to descend into the scrotum E. Twisting of the spermatic cord secondary to rotation of the left testis

This patient's presentation is consistent with a varicocele. Varicocele commonly results from blockage of blood drainage from the pampiniform plexus in the scrotum, resulting in engorgement of this venous network and the resulting classic appearance of the scrotum as a "bag of worms." Left-sided varicocele is more common due to the drainage of the left testicular vein by the left renal vein, which is susceptible to compression. This differs from the drainage of the right scrotum, where the right testicular vein drains directly into the inferior vena cava. Varicocele may also arise from idiopathic failure of valves in the testicular vein and pampiniform plexus. Boyd describes the presentation and management of varicoceles. 15% of the adult (post-pubertal) male population develops varicoceles. It is a presentation commonly associated with male infertility, with approximately one-third of men seen in fertility clinics having this lesion. Semen samples in varicocele patients may show decreased sperm counts, reduced sperm motility, and immature cellular morphology. Treatment is usually surgical; however, percutaneous approaches may be appropriate in certain situations. Shiraishi et al. review the diagnosis of varicocele and associated treatment benefits in the setting of male infertility. Surgical treatment in the form of a varicocelectomy has been shown to improve fertility with demonstrated increases in pregnancy and live-birth rates. Physical examination, scrotal ultrasound with color doppler, and scrotal temperature measurements (a surrogate marker for testicular oxidative stress) can all be used to aid in establishing a diagnosis and management strategy for a varicocele. Figure A is a clinical image of a varicocele. Illustration A depict a left-sided varicocele. Incorrect Answers: Answer 2: This mechanism is consistent with a hydrocele, which typically presents with scrotal enlargement and can be differentiated via positive transillumination test. Answer 3: This answer choice describes a case of infection of the epididymis, which presents with unilateral scrotal pain along with scrotal swelling and tenderness. Answer 4: Cryptorchidism should present as a completely absent testis or improper caudal location of the testis on physical exam of the scrotum and is not associated with the "bag of worms" appearance of the scrotum. Answer 5: This mechanism is consistent with testicular torsion, which manifests with acute onset of severe testicular pain and a tender/swollen testis.

A 71-year-old man complains of urinary hesitancy and nocturia of increasing frequency over the past several months. Digital rectal exam was positive for a slightly enlarged prostate but did not detect any additional abnormalities of the prostate or rectum. The patient's serum PSA was measured to be 6 ng/mL. Image A shows a transabdominal ultrasound of the patient. Which of the following medications should be included to optimally treat the patient's condition? A. Clonidine B. Finasteride C. Dihydrotestosterone E. Diphenhydramine D. Furosemide

This patient's presentation is consistent with benign prostatic hyperplasia (BPH). Preferred medical treatment of BPH includes an alpha-1 adrenergic blocker (ex. tamsulosin) and/or a 5-alpha reductase inhibitor (ex. finasteride). The prostate gland is located distal to the outflow tract of the bladder. Therefore, enlargement or hyperplasia of the prostate, especially in the periurethral transitional zone, presents with increased urinary hesitancy and frequency. 5-alpha reductase inhibitors act by inhibiting the conversion of testosterone to dihydrotestosterone, an important mediator of prostatic growth. Edwards describes the management of BPH. The report states that for men with mild symptoms, watchful waiting with annual reassessment is most appropriate. If needed, alpha-blockers improve symptoms relatively quickly. Comparatively, 5-alpha reductase inhibitors have a slower onset of action but may in fact alter the disease course. Transurethral resection of the prostate (TURP) often provides permanent relief but has associated surgical complications including incontinence. Cohen et al. review the efficacy of combination pharmacological treatment of BPH. Thy note that clinical trials of medical therapy for BPH have confirmed the finding that combined therapy with an a(1)-adrenergic receptor (AR) antagonist and a 5a-reductase inhibitor is superior to either agent alone. Image A is a transabdominal ultrasound demonstrating enlargement of the prostate gland as seen in BPH. Illustration A shows the progression and consequences of BPH. Illustration B is an axial CT of BPH. Notice the prostate sits posterior to the pubic symphysis and anterior to the anal canal. Incorrect Answers: Answer 1: Clonidine is an alpha-2 adrenergic agonist, which would have the undesired effect of increasing urethral resistance to flow; this agent can be used to treat stress urinary incontinence. It is not indicated in the setting of BPH. Answer 3: This would not improve BPH symptoms; in fact, it might have the opposite effect. Recall that finasteride, which prevents conversion of testosterone to dihydrotestosterone is effective in the setting of BPH. Answer 4: Antihistamines should be avoided in patients with BPH. Answer 5: As a diuretic, furosemide can worsen symptoms of BPH.

A 21-year-old man presents to the emergency department with acute back pain. The pain began a few hours prior to presentation and is located on the left lower back. The pain is described to be "shock-like," 9/10 in pain severity, and radiates to the left groin. His temperature is 98.6°F (37°C), blood pressure is 120/75 mmHg, pulse is 101/min, and respirations are 18/min. The patient appears uncomfortable and is mildly diaphoretic. There is costovertebral angle tenderness and genitourinary exam is unremarkable. A non-contrast computerized tomography (CT) scan of the abdomen and pelvis demonstrates an opaque lesion affecting the left ureter with mild hydronephrosis. Straining of the urine with urine crystal analysis is demonstrated. Which of the following amino acids is most likely poorly reabsorbed by this patient's kidney? A. Aspartic acid B. Histidine C. Isoleucine D. Lysine E. Phenylalanine

This patient's presentation is consistent with nephrolithiasis caused by cystinuria. This hereditary condition results in poor reabsorption of lysine. Filtered cystine is reabsorbed by the kidneys via a cystine transporter. This cystine transport also reabsorbes dibasic amino acids such as ornithine, arginine, and lysine. Patients with cystinuria have a defect in this transporter, resulting in increased cystine, ornithine, arginine, and lysine excretion. Ornithine, arginine, and lysine are relatively soluble enough to not result in stone formation with their increased excretion; however, cystine stones do precipitate, especially in the acidic environment of urine. Figure A demonstrates a hexagonal cystine crystal which is pathognomonic for cystinuria. Incorrect Answers: Answer 1-3 & 5: Aspartic acid, histidine, isoleucine, and phenylalanine reabsorption is not affected with cystinuria. Bullet Summary: Cystine stone formation is caused by a hereditary defect in the cystine transporter of the kidneys, which leads to impaired reabsorption of cystine, ornithine, lysine, and arginine.

A 49-year-old G4P4 woman comes to the clinic complaining of repeated leakage of fluid from her vagina for the past 5 months. She noticed an increase in episodes following her cold last week when she was coughing and sneezing a lot. Her past medical history is significant for Crohn disease, which is well controlled with sulfasalazine. Her last menstrual period was 1 year ago. She is currently sexually active with multiple partners with inconsistent condom use. She denies any vaginal itching, abnormal discharge, pain, subpubic pressure, urinary urges, or odors. Physical examination is significant for a bulge at the anterior vaginal wall. What is the most likely explanation for this patient's symptoms? A. Prolapse of the bladder B. Detrusor muscle overactivity C. Genitourinary syndrome of menopause D. Rectovaginal fistula E. Sexually transmitted infection

This patient's symptoms (presumed urine leakage following sneezing and coughing) along with her physical exam findings (anterior vaginal bulge) suggest stress urinary incontinence secondary to cystocele or prolapse of the bladder. Cystocele results from inadequate pelvic support, which is made up of the interaction between muscles of the pelvic floor and connective tissue attachments to the bony pelvis. Risk factors include increased parity, advanced age, obesity, chronic constipation, and hysterectomy. Patients most commonly present with a sensation of pelvic pressure/heaviness or "feeling a bulge." Others may experience stress urinary incontinence, which is the unintentional loss of urine usually following increases in pressure on the bladder (e.g., coughing sneezing, heavy lifting, or running.) The condition is diagnosed by pelvic examination, where a bulge can be visualized. Management includes conservative therapy (e.g., vaginal pessary or pelvic floor muscle exercises) and surgical treatments. Incorrect Answers: Answer 2: Detrusor muscle overactivity is seen in urgency incontinence, where an overactive bladder leads to urinary leaks with an urge to void immediately. Urinary leakages are not associated with activities such as sneezing or coughing. Answer 3: Genitourinary syndrome of menopause may be seen in women during menopause due to a decrease in estrogen levels. Symptoms may include an urgent need to urinate or painful urination. Although this patient is in menopause, her lack of other symptoms (e.g., vaginal dryness) and physical examination findings (e.g., vaginal bulge) makes cystocele/prolapse of the bladder more likely. Answer 4: Rectovaginal fistula (RVF) is an abnormal fistulous tract that extends between the rectum and the vagina. Patients can present with foul-smelling vaginal discharge, recurrent vaginal or urinary tract infections, and irritation or pain at the vulva/vaginal area. This patient's history of Crohn disease increases her risk of RVF. However, her lack of the aforementioned symptoms makes this less likely. Answer 5: Sexually transmitted infection (STI) is likely in this patient as she is currently sexually active with multiple partners. However, STIs rarely present with intermittent urinary leakage following increases in abdominal pressure. Bullet Summary: Cystocele, or prolapse of the bladder at or beyond the vaginal wall, can lead to stress urinary incontinence.

A 58-year-old man comes to the clinic complaining of increased urinary frequency for the past 3 days. The patient reports that he has had to get up every few hours in the night to go to the bathroom, and says "whenever I feel the urge I have to go right away." Past medical history is significant for a chlamydial infection in his twenties that was adequately treated. He endorses lower back pain and subjective warmth for the past 2 days. A rectal examination reveals a slightly enlarged prostate that is tender to palpation. What is the most likely explanation for this patient's symptoms? A. Benign prostatic hyperplasia B. Chemical irritation of the prostate C. Infection with Escherichia coli D. Prostatic adenocarcinoma E. Reinfection with Chlamydia trachomatis

This patient's urinary urgency, frequency, lower back pain, and physical examination findings (e.g., enlarged and tender prostate) suggest acute prostatitis. In men over the age of 35, acute prostatitis is most commonly due to infection with E. coli. Prostatitis is a common condition that involves inflammation of the prostate and typically affects young and middle-aged men. In men over the age of 35, it is most commonly caused by gram-negative bacteria that lead to urinary tract infections, with E. coli being the most common. In men under the age of 35, pathogens such as Neisseria gonorrhoeae and C. trachomatis are more likely. In acute bacterial prostatitis, patients present with signs of bacterial infection (e.g., fevers and chills), dysuria, pelvic/perineal pain, frequency, urgency, and lower back pain. Physical examination would produce a firm, edematous, and tender prostate. Treatment involves antimicrobial therapy such as trimethoprim-sulfamethoxazole. Incorrect Answers: Answer 1: Benign prostatic hyperplasia (BPH) can present with increased urinary frequency and urgency. This patient is at risk given his age. However, the physical examination findings (e.g., tender prostate) are more suggestive of prostatitis. Answer 2: Chemical irritation of the prostate is one of the causes of chronic prostatitis. This patient's acute onset of symptoms, along with signs of infection (e.g., subjective warmth), suggest acute prostatitis. Answer 4: Prostatic adenocarcinoma is a differential diagnosis given this patient's age and complaints of lower back pain. However, this patient's presentation of urinary symptoms and examination findings are more suggestive of prostatitis. Speaking to the patient regarding prostate cancer screening methods would be appropriate. Answer 5: Reinfection with C. trachomatis is unlikely as this patient's past infection was adequately treated. In addition, his age makes this less likely. Bullet Summary: In a patient over the age of 35 who presents with signs of prostatitis, infection with Escherichia coli is the most likely cause.

A 45-year-old Caucasian male with a history of chronic myeloid leukemia for which he is receiving chemotherapy presents to the emergency room with oliguria and colicky left flank pain. His serum creatinine is 3.0 mg/dL and is urine pH is 5.0. You diagnose nephrolithiasis. His kidney stones, however, are not visible on abdominal x-ray. His stone is most likely composed of which of the following? A. Calcium oxalate B. Calcium phosphate C. Magnesium ammonium phosphate D. Uric acid E. Cystine

Uric acid kidney stones are often associated with leukemia and myeloproliferative disorders. This patient is most likely suffering from tumor lysis syndrome, in which chemotherapy treatment causes tumor cells to lyse and release uric acid (a metabolite of tumor nucleic acid). Note that uric acid stones require an acidic environment to precipitate, while magnesium ammonium phosphate (struvite) stones require an alkaline environment to precipitate. Uric acid stones also form secondary to gout and renal tubular acidosis. Frassetto et al. review risk factors and prevention of kidney stones, which are increasingly prevalent among women and the elderly. Obesity, medications such as protease inhibitors and diuretics, high-animal protein diets, and dehydration all promote stone formation. They advocate vegetable-rich diets and supplemental citrate to prevent uric acid, calcium oxalate, and cystine stones. Mukherjee et al. present three cases of tumor lysis syndrome that resulted in renal failure despite normal uric acid levels: two from hyperphosphatemia and one from ischemic necrosis. Tumor lysis syndrome should be considered in any patient with signs of acute renal failure and high tumor burden as well as either hyperuricemia or hyperphosphatemia. Illustration A is a diagram showing the contributors to renal failure in tumor lysis syndrome. Incorrect Answers: Answer 1: Calcium oxalate stones are radiopaque (seen on x-ray). Answer 2: Calcium phosphate stones are radiopaque (seen on x-ray). Answer 3: Magnesium ammonium phosphate (struvite) stones form in alkaline, not acidic, environments. Answer 5: Cystine stones are the least common and less likely than uric acid stones in the setting of tumor lysis syndrome.

A 68-year-old woman presents to her primary care physician with leakage of urine when she sneezes, laughs, or coughs. She reports that these symptoms strictly occur during the day and never at night. She denies any subjective fever, dysuria, or hematuria. Pelvic examination is notable for a protrusion from the anterior vagina.

Urinary Incontinence (Stress incontinence) (snapshot)

A 42-year-old woman presents to her primary care physician due to involuntary passage of urine. She reports to voiding small amounts of urine throughout the day and night. She says that this occurs without warning. Medical history is significant for multiple sclerosis treated with ocrelizumab. Physical examination is notable for a distended bladder.

Urinary Incontinence(Overflow incontinence secondary to multiple sclerosis)

A 13-year-old boy presents to the urgent care clinic and reports pain in his scrotal area. He denies any rashes or inciting trauma. The pain is dull and does not radiate anywhere. On physical exam, there is a "bag of worms" on palpation of the left scrotum. The scrotum does not transilluminate with light.

VARICOCLE(SNAPSHOT)

A 34-year-old man presents to the fertility clinic for evaluation of infertility. His him and his wife have been trying to have children for 2 years. His wife was recently evaluated and found to be normal and healthy. The patient denies any pain in his testicular region. However, he reports occasional feelings of heaviness in his scrotum. On physical exam, his scrotum looks distended. Valsalva maneuvers result in a "bag of worm"-like finding upon palpation of the testicle.

Varicocele (snapshot)


Conjuntos de estudio relacionados

Financial Accounting Test 1 Multiple Choice Review

View Set

Medical-Surgical Nursing Week 3 Part 2

View Set

Advanced Research in Psychology - Exam 1

View Set

As Accounting: Limited Companies

View Set